Решить по формуле крамера систему: Метод Крамера онлайн

Содержание

Решение методом Крамера системы линейных уравнений 3-4-го порядка

Решать системы линейных алгебраических уравнений второго, третьего, изредка четвертого порядка методом Крамера достаточно часто придется студентам младших курсов учебы при изучении основ линейной алгебры. Для большинства студентов стационарной формы учебы такие задания не являются сложными, однако кто выбрал заочную учебу или дистанционную, или пропустил по определенным причинам практические занятия, вычисления выглядят непонятными и тяжелыми. Чтобы исправить такую ситуацию в данной статье будут приведены наиболее распространены примеры данной темы и схема их решения. Если Вы хорошо поймете принцип их решения, то на практике у Вас не будет трудностей с подобными заданиями.

Для начала выберем задание из сборника задач Дубовика В.П., Юрика І.І. “Высшая математика”.

———————————–

Примеры

Решить систему линейных алгебраических уравнений.

1) (1. 153)

2) (4. 165)

3) (4. 174)

Решение.

1) В случае двух уравнений решение можно получить более простым способом. Выражаемый из второго уравнения

и подставим в первое

Раскрыв скобки, сгруппируем подобные слагаемые

Отсюда получим решение

Переменнуюнайдем подстановкой в любое из уравнений

Таким образом решением системы двух уравнений будут следующие значения

Поскольку цель статьи научить студентов решать по методике Крамера то решим данный пример и етим методом.

Для этого выпишем систему линейных уравнений в виде

Найдем детерминант основной части

Для вычисления вспомогательных определителей ставим столбец свободных членов на место первой строки для и на место второй для . В результате получим

Подставим найденные значения в формулы Крамера

и найдем неизвестные

Из рассмотренного примера видим что вычисление при двух уравнениях с двумя неизвестными достаточно простые.

2) Запишем систему трех алгебраических уравнений в удобном для решения виде

Найдем детерминант системы по правилу треугольников

Для вычисления дополнительных определителей подставляем столбец свободных членов на место первого, второго и третьего столбцов. В результате получим

Вычисляем неизвестные за формулами Крамера

Для данного примера нахождения решения также не слишком сложно, хотя по сравнению с системой двух уравнений вычислений заметно прибавилось.

3) Записываем систему уравнений четвертого порядка в виде

Находим главный определитель системы. При вычислении детерминантов четвертого порядка их необходимо раскладывать за строками или столбцами у каторых больше всего нулей. Поскольку в данном случае нулей главный определитель не имеет то разложим его за первой строкой

и найдем соответствующие детермиінанты третьего порядка

Подставим найденные значения в определитель

По такой же схеме вычисляем вспомогательные определители, напомню лишь, что они образуются заменой столбца в главном определителе на столбец свободных членов (обозначен черным цветом). Я не буду приводить детальных излаганий, однако Вы можете проверить, что детерминанты примут значение

Подставив в формулы Крамера, после вычислений будем иметь

На этом пример решено.

Системы четырех линейных уравнений наиболее трудоемкие в вычислениях, для вычисления их решения нужно решать 5*4 определители третьего порядка, в то время как системы трех уравнений лиш 4. Будьте внимательные при вычислениях ведь самая малая ошибка может иметь следствием неверный результат.

———————————————-

Посмотреть материалы:

Решение систем уравнений с помощью формул крамера. Метод крамера решения систем линейных уравнений

Методы Крамера и Гаусса – одни из самых популярных методов решения СЛАУ . К тому же, в ряде случаев целесообразно использовать именно конкретные методы. Сессия близка, и сейчас самое время повторить или освоить их с нуля. Сегодня разбираемся с решением методом Крамера. Ведь решение системы линейных уравнений методом Крамера – весьма полезный навык.

Системы линейных алгебраических уравнений

Система линейных алгебраических уравнений – система уравнений вида:

Набор значений x , при котором уравнения системы обращаются в тождества, называется решением системы, a и b – вещественные коэффициенты. Простенькую систему, состоящую из двух уравнений с двумя неизвестными, можно решить в уме либо выразив одну переменную через другую. Но переменных (иксов) в СЛАУ может быть гораздо больше двух, и здесь простыми школьными манипуляциями не обойтись. Что же делать? Например, решать СЛАУ методом Крамера!

Итак, пусть система состоит из n уравнений с n неизвестными.

Такую систему можно переписать в матричном виде

Здесь A – основная матрица системы, X и B , соответственно, матрицы-столбцы неизвестных переменных и свободных членов.

Решение СЛАУ методом Крамера

Если определитель главной матрицы не равен нулю (матрица невырожденная), систему можно решать по методу Крамера.

Согласно методу Крамера, решение находится по формулам:

Здесь дельта – определитель главной матрицы, а дельта x n-ное – определитель, полученный из определителя главной матрицы путем заменой n-ного столбца на столбец свободных членов.

В этом и заключается вся суть метода Крамера. Подставляя найденные по вышеприведенным формулам значения x в искомую систему, убеждаемся в правильности (или наоборот) нашего решения. Чтобы Вы быстрее уловили суть, приведем ниже пример подробного решения СЛАУ методом Крамера:

Даже если у Вас не получится с первого раза, не расстраивайтесь! Немного практики, и Вы начнете щелкать СЛАУ как орешки. Более того, сейчас совершенно необязательно корпеть над тетрадью, решая громоздкие выкладки и исписывая стержень. Можно легко решить СЛАУ методом Крамера в режиме онлайн, лишь подставив в готовую форму коэффициенты. Испробовать онлайн калькулятор решения методом Крамера можно, к примеру, на этом сайте .


А если система оказалась упорной и не сдается, Вы всегда можете обратиться за помощью к нашим авторам, например, чтобы . Будь в системе хоть 100 неизвестных, мы обязательно решим ее верно и точно в срок!

В первой части мы рассмотрели немного теоретического материала, метод подстановки, а также метод почленного сложения уравнений системы. Всем, кто зашел на сайт через эту страницу рекомендую ознакомиться с первой частью. Возможно, некоторым посетителям покажется материал слишком простым, но по ходу решения систем линейных уравнений я сделал ряд очень важных замечаний и выводов, касающихся решения математических задач в целом.

А сейчас мы разберём правило Крамера, а также решение системы линейных уравнений с помощью обратной матрицы (матричный метод). Все материалы изложены просто, подробно и понятно, практически все читатели смогут научиться решать системы вышеуказанными способами.

Сначала мы подробно рассмотрим правило Крамера для системы двух линейных уравнений с двумя неизвестными. Зачем? – Ведь простейшую систему можно решить школьным методом, методом почленного сложения!

Дело в том, что пусть иногда, но встречается такое задание – решить систему двух линейных уравнений с двумя неизвестными по формулам Крамера. Во-вторых, более простой пример поможет понять, как использовать правило Крамера для более сложного случая – системы трех уравнений с тремя неизвестными.

Кроме того, существуют системы линейных уравнений с двумя переменными, которые целесообразно решать именно по правилу Крамера!

Рассмотрим систему уравнений

На первом шаге вычислим определитель , его называют главным определителем системы .

метод Гаусса .

Если , то система имеет единственное решение, и для нахождения корней мы должны вычислить еще два определителя:
и

На практике вышеуказанные определители также могут обозначаться латинской буквой .

Корни уравнения находим по формулам:
,

Пример 7

Решить систему линейных уравнений

Решение : Мы видим, что коэффициенты уравнения достаточно велики, в правой части присутствуют десятичные дроби с запятой. Запятая – довольно редкий гость в практических заданиях по математике, эту систему я взял из эконометрической задачи.

Как решить такую систему? Можно попытаться выразить одну переменную через другую, но в этом случае наверняка получатся страшные навороченные дроби, с которыми крайне неудобно работать, да и оформление решения будет выглядеть просто ужасно. Можно умножить второе уравнение на 6 и провести почленное вычитание, но и здесь возникнут те же самые дроби.

Что делать? В подобных случаях и приходят на помощь формулы Крамера.

;

;

Ответ : ,

Оба корня обладают бесконечными хвостами, и найдены приближенно, что вполне приемлемо (и даже обыденно) для задач эконометрики.

Комментарии здесь не нужны, поскольку задание решается по готовым формулам, однако, есть один нюанс. Когда используете данный метод, обязательным фрагментом оформления задания является следующий фрагмент: «, значит, система имеет единственное решение» . В противном случае рецензент может Вас наказать за неуважение к теореме Крамера.

Совсем не лишней будет проверка, которую удобно провести на калькуляторе: подставляем приближенные значения в левую часть каждого уравнения системы. В результате с небольшой погрешностью должны получиться числа, которые находятся в правых частях.

Пример 8

Ответ представить в обыкновенных неправильных дробях. Сделать проверку.

Это пример для самостоятельного решения (пример чистового оформления и ответ в конце урока).

Переходим к рассмотрению правила Крамера для системы трех уравнений с тремя неизвестными:

Находим главный определитель системы:

Если , то система имеет бесконечно много решений или несовместна (не имеет решений). В этом случае правило Крамера не поможет, нужно использовать метод Гаусса .

Если , то система имеет единственное решение и для нахождения корней мы должны вычислить еще три определителя:
, ,

И, наконец, ответ рассчитывается по формулам:

Как видите, случай «три на три» принципиально ничем не отличается от случая «два на два», столбец свободных членов последовательно «прогуливается» слева направо по столбцам главного определителя.

Пример 9

Решить систему по формулам Крамера.

Решение : Решим систему по формулам Крамера.

, значит, система имеет единственное решение.

Ответ : .

Собственно, здесь опять комментировать особо нечего, ввиду того, что решение проходит по готовым формулам. Но есть пара замечаний.

Бывает так, что в результате вычислений получаются «плохие» несократимые дроби, например: .

Я рекомендую следующий алгоритм «лечения». Если под рукой нет компьютера, поступаем так:

1) Возможно, допущена ошибка в вычислениях. Как только Вы столкнулись с «плохой» дробью, сразу необходимо проверить, правильно ли переписано условие . Если условие переписано без ошибок, то нужно пересчитать определители, используя разложение по другой строке (столбцу).

2) Если в результате проверки ошибок не выявлено, то вероятнее всего, допущена опечатка в условии задания. В этом случае спокойно и ВНИМАТЕЛЬНО прорешиваем задание до конца, а затем обязательно делаем проверку и оформляем ее на чистовике после решения. Конечно, проверка дробного ответа – занятие неприятное, но зато будет обезоруживающий аргумент для преподавателя, который ну очень любит ставить минус за всякую бяку вроде . Как управляться с дробями, подробно расписано в ответе для Примера 8.

Если под рукой есть компьютер, то для проверки используйте автоматизированную программу, которую можно бесплатно скачать в самом начале урока. Кстати, выгоднее всего сразу воспользоваться программой (еще до начала решения), Вы сразу будете видеть промежуточный шаг, на котором допустили ошибку! Этот же калькулятор автоматически рассчитывает решение системы матричным методом.

Замечание второе. Время от времени встречаются системы в уравнениях которых отсутствуют некоторые переменные, например:

Здесь в первом уравнении отсутствует переменная , во втором – переменная . В таких случаях очень важно правильно и ВНИМАТЕЛЬНО записать главный определитель:

– на месте отсутствующих переменных ставятся нули.
Кстати определители с нулями рационально раскрывать по той строке (столбцу), в которой находится ноль, так как вычислений получается заметно меньше.

Пример 10

Решить систему по формулам Крамера.

Это пример для самостоятельного решения (образец чистового оформления и ответ в конце урока).

Для случая системы 4 уравнений с 4 неизвестными формулы Крамера записываются по аналогичным принципам. Живой пример можно посмотреть на уроке Свойства определителя. Понижение порядка определителя – пять определителей 4-го порядка вполне решабельны. Хотя задача уже весьма напоминает ботинок профессора на груди у студента-счастливчика.

Решение системы с помощью обратной матрицы

Метод обратной матрицы – это, по существу, частный случай матричного уравнения (см. Пример №3 указанного урока).

Для изучения данного параграфа необходимо уметь раскрывать определители, находить обратную матрицу и выполнять матричное умножение. Соответствующие ссылки будут даны по ходу объяснений.

Пример 11

Решить систему с матричным методом

Решение : Запишем систему в матричной форме:
, где

Пожалуйста, посмотрите на систему уравнений и на матрицы. По какому принципу записываем элементы в матрицы, думаю, всем понятно. Единственный комментарий: если бы в уравнениях отсутствовали некоторые переменные, то на соответствующих местах в матрице нужно было бы поставить нули.

Обратную матрицу найдем по формуле:
, где – транспонированная матрица алгебраических дополнений соответствующих элементов матрицы .

Сначала разбираемся с определителем:

Здесь определитель раскрыт по первой строке.

Внимание! Если , то обратной матрицы не существует, и решить систему матричным методом невозможно. В этом случае система решается методом исключения неизвестных (методом Гаусса) .

Теперь нужно вычислить 9 миноров и записать их в матрицу миноров

Справка: Полезно знать смысл двойных подстрочных индексов в линейной алгебре. Первая цифра – это номер строки, в которой находится данный элемент. Вторая цифра – это номер столбца, в котором находится данный элемент:

То есть, двойной подстрочный индекс указывает, что элемент находится в первой строке, третьем столбце, а, например, элемент находится в 3 строке, 2 столбце


2. Решение систем уравнений матричным методом (при помощи обратной матрицы).
3. Метод Гаусса решения систем уравнений.

Метод Крамера.

Метод Крамера применяется для решения систем линейных алгебраических уравнений (СЛАУ ).

Формулы на примере системы из двух уравнений с двумя переменными.
Дано: Решить методом Крамера систему

Относительно переменных х и у .
Решение:
Найдем определитель матрицы, составленный из коэффициентов системы Вычисление определителей. :



Применим формулы Крамера и найдем значения переменных:
и .
Пример 1:
Решить систему уравнений:

относительно переменных х и у .
Решение:


Заменим в этом определителе первый столбец столбцом коэффициентов из правой части системы и найдем его значение:

Сделаем аналогичное действие, заменив в первом определителе второй столбец:

Применим формулы Крамера и найдем значения переменных:
и .
Ответ:
Замечание: Этим методом можно решать системы и большей размерности.

Замечание: Если получается, что , а делить на ноль нельзя, то говорят, что система не имеет единственного решения. В этом случае система имеет или бесконечно много решений или не имеет решений вообще.

Пример 2 (бесконечное количество решений):

Решить систему уравнений:

относительно переменных х и у .
Решение:
Найдем определитель матрицы, составленный из коэффициентов системы:

Решение систем методом подстановки.

Первое из уравнений системы — равенство, верное при любых значениях переменных (потому что 4 всегда равно 4). Значит, остается только одно уравнение. Это уравнение связи между переменными .
Получили, решением системы являются любые пары значений переменных, связанных между собой равенством .
Общее решение запишется так:
Частные решения можно определять выбирая произвольное значение у и вычисляя х по этому равенству связи.

и т.д.
Таких решений бесконечно много.
Ответ: общее решение
Частные решения:

Пример 3 (решений нет, система несовместна):

Решить систему уравнений:

Решение:
Найдем определитель матрицы, составленный из коэффициентов системы:

Применять формулы Крамера нельзя. Решим эту систему методом подстановки

Второе уравнение системы — равенство, неверное ни при каких значениях переменных (конечно же, так как -15 не равно 2). Если одно из уравнений системы не верно ни при каких значениях переменных, то и вся системы не имеет решений.
Ответ: решений нет

Пусть система линейных уравнений содержит столько уравнений, каково количество независимых переменных, т.е. имеет вид

Такие системы линейных уравнений называются квадратными. Определитель, составленный из коэффициентов при независимых переменных системы (1.5), называется главным определителем системы. Мы будем обозначать его греческой буквой D. Таким образом,

. (1.6)

Если в главном определителе произвольный (j -ый) столбец, заменить столбцом свободных членов системы (1.5), то можно получить еще n вспомогательных определителей:

(j = 1, 2, …, n ). (1.7)

Правило Крамера решения квадратных систем линейных уравнений заключается в следующем. Если главный определитель D системы (1.5) отличен от нуля, то система имеет и притом единственное решение, которое можно найти по формулам:

(1.8)

Пример 1.5. Методом Крамера решить систему уравнений

.

Вычислим главный определитель системы:

Так как D¹0, то система имеет единственное решение, которое можно найти по формулам (1.8):

Таким образом,

Действия над матрицами

1. Умножение матрицы на число. Операция умножения матрицы на число определяется следующим образом.

2. Для того чтобы умножить матрицу на число, нужно все ее элементы умножить на это число. То есть

. (1.9)

Пример 1.6. .

Сложение матриц.

Данная операция вводится только для матриц одного и того же порядка.

Для того чтобы сложить две матрицы, необходимо к элементам одной матрицы прибавить соответствующие элементы другой матрицы:

(1.10)
Операция сложения матриц обладает свойствами ассоциативности и коммутативности.

Пример 1.7. .

Умножение матриц.

Если число столбцов матрицы А совпадает с числом строк матрицы В , то для таких матриц вводится операция умножения:

2

Таким образом, при умножении матрицы А размерности m ´n на матрицу В размерности n ´k мы получаем матрицу С размерности m ´k . При этом элементы матрицы С вычисляются по следующим формулам:

Задача 1.8. Найти, если это возможно, произведение матриц AB и BA :

Решение. 1) Для того чтобы найти произведение AB , необходимо строки матрицы A умножить на столбцы матрицы B :

2) Произведение BA не существует, т. к. количество столбцов матрицы B не совпадает с количеством строк матрицы A .

Обратная матрица. Решение систем линейных уравнений матричным способом

Матрица A – 1 называется обратной к квадратной матрице А , если выполнено равенство:

где через I обозначается единичная матрица того же порядка, что и матрица А :

.

Для того чтобы квадратная матрица имела обратную необходимо и достаточно, чтобы ее определитель был отличен от нуля. Обратную матрицу находят по формуле:

, (1.13)

где A ij – алгебраические дополнения к элементам a ij матрицы А (заметим, что алгебраические дополнения к строкам матрицы А располагаются в обратной матрице в виде соответствующих столбцов).

Пример 1.9. Найти обратную матрицу A – 1 к матрице

.

Обратную матрицу найдем по формуле (1.13), которая для случая n = 3 имеет вид:

.

Найдем det A = | A | = 1 × 3 × 8 + 2 × 5 × 3 + 2 × 4 × 3 – 3 × 3 × 3 – 1 × 5 × 4 – 2 × 2 × 8 = 24 + 30 + 24 – 27 – 20 – 32 = – 1. Так как определитель исходной матрицы отличен от нуля, то обратная матрица существует.

1) Найдем алгебраические дополнения A ij :

Для удобства нахождения обратной матрицы, алгебраические дополнения к строкам исходной матрицы мы расположили в соответствующие столбцы.

Из полученных алгебраических дополнений составим новую матрицу и разделим ее на определитель det A . Таким образом, мы получим обратную матрицу:

Квадратные системы линейных уравнений с отличным от нуля главным определителем можно решать с помощью обратной матрицы. Для этого систему (1.5) записывают в матричном виде:

где

Умножая обе части равенства (1.14) слева на A – 1 , мы получим решение системы:

, откуда

Таким образом, для того чтобы найти решение квадратной системы, нужно найти обратную матрицу к основной матрице системы и умножить ее справа на матрицу-столбец свободных членов.

Задача 1.10. Решить систему линейных уравнений

с помощью обратной матрицы.

Решение. Запишем систему в матричном виде: ,

где – основная матрица системы, – столбец неизвестных и – столбец свободных членов. Так как главный определитель системы , то основная матрица системы А имеет обратную матрицу А -1 . Для нахождения обратной матрицы А -1 , вычислим алгебраические дополнения ко всем элементам матрицы А :

Из полученных чисел составим матрицу (причем алгебраические дополнения к строкам матрицы А запишем в соответствующие столбцы) и разделим ее на определитель D. Таким образом, мы нашли обратную матрицу:

Решение системы находим по формуле (1.15):

Таким образом,

Решение систем линейных уравнений методом обыкновенных жордановых исключений

Пусть дана произвольная (не обязательно квадратная) система линейных уравнений:

(1.16)

Требуется найти решение системы, т.е. такой набор переменных , который удовлетворяет всем равенствам системы (1.16). В общем случае система (1.16) может иметь не только одно решение, но и бесчисленное множество решений. Она может так же вообще не иметь решений.

При решении подобных задач используется хорошо известный из школьного курса метод исключения неизвестных, который еще называется методом обыкновенных жордановых исключений. Суть данного метода заключается в том, что в одном из уравнений системы (1.16) одна из переменных выражается через другие переменные. Затем эта переменная подставляется в другие уравнения системы. В результате получается система, содержащая на одно уравнение и на одну переменную меньше, чем исходная система. Уравнение, из которого выражалась переменная, запоминается.

Этот процесс повторяется до тех пор, пока в системе не останется одно последнее уравнение. В процессе исключения неизвестных некоторые уравнения могут превратиться в верные тождества, например . Такие уравнения из системы исключаются, так как они выполняются при любых значениях переменных и, следовательно, не оказывают влияния на решение системы. Если в процессе исключения неизвестных хотя бы одно уравнение становится равенством, которое не может выполняться ни при каких значениях переменных (например ), то мы делаем вывод, что система не имеет решения.

Если в ходе решения противоречивых уравнений не возникло, то из последнего уравнения находится одна из оставшихся в нем переменных. Если в последнем уравнении осталась только одна переменная, то она выражается числом. Если в последнем уравнении остаются еще и другие переменные, то они считаются параметрами, и выраженная через них переменная будет функцией этих параметров. Затем совершается так называемый «обратный ход». Найденную переменную подставляют в последнее запомненное уравнение и находят вторую переменную. Затем две найденные переменные подставляют в предпоследнее запомненное уравнение и находят третью переменную, и так далее, вплоть до первого запомненного уравнения.

В результате мы получаем решение системы. Данное решение будет являться единственным, если найденные переменные будут числами. Если же первая найденная переменная, а затем и все остальные будут зависеть от параметров, то система будет иметь бесчисленное множество решений (каждому набору параметров соответствует новое решение). Формулы, позволяющие найти решение системы в зависимости от того или иного набора параметров, называются общим решением системы.

Пример 1.11.

x

После запоминания первого уравнения и приведения подобных членов во втором и третьем уравнении мы приходим к системе:

Выразим y из второго уравнения и подставим его в первое уравнение:

Запомним второе уравнение, а из первого найдем z :

Совершая обратный ход, последовательно найдем y и z . Для этого сначала подставим в последнее запомненное уравнение , откуда найдем y :

.

Затем подставим и в первое запомненное уравнение , откуда найдем x :

Задача 1.12. Решить систему линейных уравнений методом исключения неизвестных:

. (1.17)

Решение. Выразим из первого уравнения переменную x и подставим ее во второе и третье уравнения:

.

Запомним первое уравнение

В данной системе первое и второе уравнения противоречат друг другу. Действительно, выражая y , получим, что 14 = 17. Данное равенство не выполняется, ни при каких значениях переменных x , y , и z . Следовательно, система (1.17) несовместна, т.е. не имеет решения.

Читателям предлагаем самостоятельно проверить, что главный определитель исходной системы (1.17) равен нулю.

Рассмотрим систему, отличающуюся от системы (1.17) всего лишь одним свободным членом.

Задача 1.13. Решить систему линейных уравнений методом исключения неизвестных:

. (1.18)

Решение. Как и прежде, выразим из первого уравнения переменную x и подставим ее во второе и третье уравнения:

.

Запомним первое уравнение и приведем подобные члены во втором и третьем уравнении. Мы приходим к системе:

Выражая y из первого уравнения и подставляя его во второе уравнение , мы получим тождество 14 = 14, которое не влияет на решение системы, и, следовательно, его можно из системы исключить.

В последнем запомненном равенстве переменную z будем считать параметром. Полагаем . Тогда

Подставим y и z в первое запомненное равенство и найдем x :

.

Таким образом, система (1.18) имеет бесчисленное множество решений, причем любое решение можно найти по формулам (1.19), выбирая произвольное значение параметра t :

(1.19)
Так решениями системы, например, являются следующие наборы переменных (1; 2; 0), (2; 26; 14) и т. д. Формулы (1.19) выражают общее (любое) решение системы (1.18).

В том случае, когда исходная система (1.16) имеет достаточно большое количество уравнений и неизвестных, указанный метод обыкновенных жордановых исключений представляется громоздким. Однако это не так. Достаточно вывести алгоритм пересчета коэффициентов системы при одном шаге в общем виде и оформить решение задачи в виде специальных жордановых таблиц.

Пусть дана система линейных форм (уравнений):

, (1.20)
где x j – независимые (искомые) переменные, a ij – постоянные коэффициенты
(i = 1, 2,…, m ; j = 1, 2,…, n ). Правые части системы y i (i = 1, 2,…, m ) могут быть как переменными (зависимыми), так и константами. Требуется найти решений данной системы методом исключения неизвестных.

Рассмотрим следующую операцию, называемую в дальнейшем «одним шагом обыкновенных жордановых исключений». Из произвольного (r -го) равенства выразим произвольную переменную (x s ) и подставим во все остальные равенства. Разумеется, это возможно только в том случае, когда a rs ¹ 0. Коэффициент a rs называется разрешающим (иногда направляющим или главным) элементом.

Мы получим следующую систему:

. (1.21)

Из s -го равенства системы (1.21) мы впоследствии найдем переменную x s (после того, как будут найдены остальные переменные). S -я строка запоминается и в дальнейшем из системы исключается. Оставшаяся система будет содержать на одно уравнение и на одну независимую переменную меньше, чем исходная система.

Вычислим коэффициенты полученной системы (1.21) через коэффициенты исходной системы (1.20). Начнем с r -го уравнения, которое после выражения переменной x s через остальные переменные будет выглядеть следующим образом:

Таким образом, новые коэффициенты r -го уравнения вычисляются по следующим формулам:

(1.23)
Вычислим теперь новые коэффициенты b ij (i ¹ r ) произвольного уравнения. Для этого подставим выраженную в (1.22) переменную x s в i -е уравнение системы (1.20):

После приведения подобных членов, получим:

(1.24)
Из равенства (1.24) получим формулы, по которым вычисляются остальные коэффициенты системы (1.21) (за исключением r -го уравнения):

(1.25)
Преобразование систем линейных уравнений методом обыкновенных жордановых исключений оформляется в виде таблиц (матриц). Эти таблицы получили название «жордановых».

Так, задаче (1.20) ставится в соответствие следующая жорданова таблица:

Таблица 1.1

x 1x 2x j x s x n
y 1 =a 11a 12a 1j a 1s a 1n
…………………………………………………………………..
y i =a i 1a i 2a ij a is a in
…………………………………………………………………..
y r =a r 1a r 2a rj a rsa rn
………………………………………………………………….
y n =a m 1a m 2a mj a ms a mn

Жорданова таблица 1.1 содержит левый заглавный столбец, в который записывают правые части системы (1.20) и верхнюю заглавную строку, в которую записывают независимые переменные.

Остальные элементы таблицы образуют основную матрицу коэффициентов системы (1.20). Если умножить матрицу А на матрицу , состоящую из элементов верхней заглавной строки, то получится матрица , состоящая из элементов левого заглавного столбца. То есть, по существу, жорданова таблица это матричная форма записи системы линейных уравнений: . Системе (1.21) при этом соответствует следующая жорданова таблица:

Таблица 1.2

x 1x 2x j y r x n
y 1 =b 11b 12b 1 j b 1 s b 1 n
…………………………………………………………………..
y i = b i 1b i 2b ij b is b in
…………………………………………………………………..
x s = b r 1b r 2b rj b rs b rn
………………………………………………………………….
y n = b m 1b m 2b mj b ms b mn

Разрешающий элемент a rs мы будем выделять жирным шрифтом. Напомним, что для осуществления одного шага жордановых исключений разрешающий элемент должен быть отличен от нуля. Строку таблицы, содержащую разрешающий элемент, называют разрешающей строкой. Столбец, содержащий разрешающий элемент, называют разрешающим столбцом. При переходе от данной таблицы к следующей таблице одна переменная (x s ) из верней заглавной строки таблицы перемещается в левый заглавный столбец и, наоборот, один из свободных членов системы (y r ) из левого заглавного столбца таблицы перемещается в верхнюю заглавную строку.

Опишем алгоритм пересчета коэффициентов при переходе от жордановой таблицы (1.1) к таблице (1.2), вытекающий из формул (1.23) и (1.25).

1. Разрешающий элемент заменяется обратным числом:

2. Остальные элементы разрешающей строки делятся на разрешающий элемент и изменяют знак на противоположный:

3. Остальные элементы разрешающего столбца делятся на разрешающий элемент:

4. Элементы, не попавшие в разрешающую строку и разрешающий столбец, пересчитываются по формулам:

Последняя формула легко запоминается, если заметить, что элементы, составляющие дробь , находятся на пересечении i -ой и r -ой строк и j -го и s -го столбцов (разрешающей строки, разрешающего столбца и той строки и столбца, на пересечении которых находится пересчитываемый элемент). Точнее, при запоминании формулы можно использовать следующую диаграмму:

-21-26-13-37

Совершая первый шаг жордановых исключений, в качестве разрешающего элемента можно выбрать любой элемент таблицы 1.3, расположенный в столбцах x 1 ,…, x 5 (все указанные элементы не равны нулю). Не следует только выбирать разрешающий элемент в последнем столбце, т.к. требуется находить независимые переменные x 1 ,…, x 5 . Выбираем, например, коэффициент 1 при переменной x 3 в третьей строке таблицы 1.3 (разрешающий элемент показан жирным шрифтом). При переходе к таблице 1.4 переменная x 3 из верхней заглавной строки меняется местами с константой 0 левого заглавного столбца (третья строка). При этом переменная x 3 выражается через остальные переменные.

Строку x 3 (табл.1.4) можно, предварительно запомнив, исключить из таблицы 1.4. Из таблицы 1.4 исключается так же третий столбец с нулем в верхней заглавной строке. Дело в том, что независимо от коэффициентов данного столбца b i 3 все соответствующие ему слагаемые каждого уравнения 0·b i 3 системы будут равны нулю. Поэтому указанные коэффициенты можно не вычислять. Исключив одну переменную x 3 и запомнив одно из уравнений, мы приходим к системе, соответствующей таблице 1.4 (с вычеркнутой строкой x 3). Выбирая в таблице 1.4 в качестве разрешающего элемента b 14 = -5, переходим к таблице 1.5. В таблице 1.5 запоминаем первую строку и исключаем ее из таблицы вместе с четвертым столбцом (с нулем наверху).

Таблица 1.5 Таблица 1.6

Из последней таблицы 1.7 находим: x 1 = – 3 + 2x 5 .

Последовательно подставляя уже найденные переменные в запомненные строки, находим остальные переменные:

Таким образом, система имеет бесчисленное множество решений. Переменной x 5 , можно придавать произвольные значения. Данная переменная выступает в роли параметра x 5 = t. Мы доказали совместность системы и нашли ее общее решение:

x 1 = – 3 + 2t

x 2 = – 1 – 3t

x 3 = – 2 + 4t . (1.27)
x 4 = 4 + 5t

x 5 = t

Придавая параметру t различные значения, мы получим бесчисленное множество решений исходной системы. Так, например, решением системы является следующий набор переменных (- 3; – 1; – 2; 4; 0).

Для того чтобы освоить данный параграф Вы должны уметь раскрывать определители «два на два» и «три на три». Если с определителями плохо, пожалуйста, изучите урок Как вычислить определитель?

Сначала мы подробно рассмотрим правило Крамера для системы двух линейных уравнений с двумя неизвестными. Зачем? – Ведь простейшую систему можно решить школьным методом, методом почленного сложения!

Дело в том, что пусть иногда, но встречается такое задание – решить систему двух линейных уравнений с двумя неизвестными по формулам Крамера. Во-вторых, более простой пример поможет понять, как использовать правило Крамера для более сложного случая – системы трех уравнений с тремя неизвестными.

Кроме того, существуют системы линейных уравнений с двумя переменными, которые целесообразно решать именно по правилу Крамера!

Рассмотрим систему уравнений

На первом шаге вычислим определитель , его называют главным определителем системы .

метод Гаусса .

Если , то система имеет единственное решение, и для нахождения корней мы должны вычислить еще два определителя:
и

На практике вышеуказанные определители также могут обозначаться латинской буквой .

Корни уравнения находим по формулам:
,

Пример 7

Решить систему линейных уравнений

Решение : Мы видим, что коэффициенты уравнения достаточно велики, в правой части присутствуют десятичные дроби с запятой. Запятая – довольно редкий гость в практических заданиях по математике, эту систему я взял из эконометрической задачи.

Как решить такую систему? Можно попытаться выразить одну переменную через другую, но в этом случае наверняка получатся страшные навороченные дроби, с которыми крайне неудобно работать, да и оформление решения будет выглядеть просто ужасно. Можно умножить второе уравнение на 6 и провести почленное вычитание, но и здесь возникнут те же самые дроби.

Что делать? В подобных случаях и приходят на помощь формулы Крамера.

;

;

Ответ : ,

Оба корня обладают бесконечными хвостами, и найдены приближенно, что вполне приемлемо (и даже обыденно) для задач эконометрики.

Комментарии здесь не нужны, поскольку задание решается по готовым формулам, однако, есть один нюанс. Когда используете данный метод, обязательным фрагментом оформления задания является следующий фрагмент: «, значит, система имеет единственное решение» . В противном случае рецензент может Вас наказать за неуважение к теореме Крамера.

Совсем не лишней будет проверка, которую удобно провести на калькуляторе: подставляем приближенные значения в левую часть каждого уравнения системы. В результате с небольшой погрешностью должны получиться числа, которые находятся в правых частях.

Пример 8

Ответ представить в обыкновенных неправильных дробях. Сделать проверку.

Это пример для самостоятельного решения (пример чистового оформления и ответ в конце урока).

Переходим к рассмотрению правила Крамера для системы трех уравнений с тремя неизвестными:

Находим главный определитель системы:

Если , то система имеет бесконечно много решений или несовместна (не имеет решений). В этом случае правило Крамера не поможет, нужно использовать метод Гаусса .

Если , то система имеет единственное решение и для нахождения корней мы должны вычислить еще три определителя:
, ,

И, наконец, ответ рассчитывается по формулам:

Как видите, случай «три на три» принципиально ничем не отличается от случая «два на два», столбец свободных членов последовательно «прогуливается» слева направо по столбцам главного определителя.

Пример 9

Решить систему по формулам Крамера.

Решение : Решим систему по формулам Крамера.

, значит, система имеет единственное решение.

Ответ : .

Собственно, здесь опять комментировать особо нечего, ввиду того, что решение проходит по готовым формулам. Но есть пара замечаний.

Бывает так, что в результате вычислений получаются «плохие» несократимые дроби, например: .
Я рекомендую следующий алгоритм «лечения». Если под рукой нет компьютера, поступаем так:

1) Возможно, допущена ошибка в вычислениях. Как только Вы столкнулись с «плохой» дробью, сразу необходимо проверить, правильно ли переписано условие . Если условие переписано без ошибок, то нужно пересчитать определители, используя разложение по другой строке (столбцу).

2) Если в результате проверки ошибок не выявлено, то вероятнее всего, допущена опечатка в условии задания. В этом случае спокойно и ВНИМАТЕЛЬНО прорешиваем задание до конца, а затем обязательно делаем проверку и оформляем ее на чистовике после решения. Конечно, проверка дробного ответа – занятие неприятное, но зато будет обезоруживающий аргумент для преподавателя, который ну очень любит ставить минус за всякую бяку вроде . Как управляться с дробями, подробно расписано в ответе для Примера 8.

Если под рукой есть компьютер, то для проверки используйте автоматизированную программу, которую можно бесплатно скачать в самом начале урока. Кстати, выгоднее всего сразу воспользоваться программой (еще до начала решения), Вы сразу будете видеть промежуточный шаг, на котором допустили ошибку! Этот же калькулятор автоматически рассчитывает решение системы матричным методом.

Замечание второе. Время от времени встречаются системы в уравнениях которых отсутствуют некоторые переменные, например:

Здесь в первом уравнении отсутствует переменная , во втором – переменная . В таких случаях очень важно правильно и ВНИМАТЕЛЬНО записать главный определитель:
– на месте отсутствующих переменных ставятся нули.
Кстати определители с нулями рационально раскрывать по той строке (столбцу), в которой находится ноль, так как вычислений получается заметно меньше.

Пример 10

Решить систему по формулам Крамера.

Это пример для самостоятельного решения (образец чистового оформления и ответ в конце урока).

Для случая системы 4 уравнений с 4 неизвестными формулы Крамера записываются по аналогичным принципам. Живой пример можно посмотреть на уроке Свойства определителя. Понижение порядка определителя – пять определителей 4-го порядка вполне решабельны. Хотя задача уже весьма напоминает ботинок профессора на груди у студента-счастливчика.


Решение системы с помощью обратной матрицы

Метод обратной матрицы – это, по существу, частный случай матричного уравнения (см. Пример №3 указанного урока).

Для изучения данного параграфа необходимо уметь раскрывать определители, находить обратную матрицу и выполнять матричное умножение. Соответствующие ссылки будут даны по ходу объяснений.

Пример 11

Решить систему с матричным методом

Решение : Запишем систему в матричной форме:
, где

Пожалуйста, посмотрите на систему уравнений и на матрицы. По какому принципу записываем элементы в матрицы, думаю, всем понятно. Единственный комментарий: если бы в уравнениях отсутствовали некоторые переменные, то на соответствующих местах в матрице нужно было бы поставить нули.

Обратную матрицу найдем по формуле:
, где – транспонированная матрица алгебраических дополнений соответствующих элементов матрицы .

Сначала разбираемся с определителем:

Здесь определитель раскрыт по первой строке.

Внимание! Если , то обратной матрицы не существует, и решить систему матричным методом невозможно. В этом случае система решается методом исключения неизвестных (методом Гаусса) .

Теперь нужно вычислить 9 миноров и записать их в матрицу миноров

Справка: Полезно знать смысл двойных подстрочных индексов в линейной алгебре. Первая цифра – это номер строки, в которой находится данный элемент. Вторая цифра – это номер столбца, в котором находится данный элемент:

То есть, двойной подстрочный индекс указывает, что элемент находится в первой строке, третьем столбце, а, например, элемент находится в 3 строке, 2 столбце

В ходе решения расчет миноров лучше расписать подробно, хотя, при определенном опыте их можно приноровиться считать с ошибками устно.

Теорема Крамера

Системы линейных алгебраических уравнений При решении систем линейных уравнений обсуждаются 3 вопроса: а) существует ли решение системы уравнений, б) сколько разных решений имеет система уравнений, в) алгоритм решения. Ниже излагаются основные результаты в этой области математики, позволяющие исчерпывающим образом ответить на эти вопросы.

Теорема Крамера

Система двух уравнений, два неизвестных

Рассмотрим систему линейных алгебраических уравнений \[ a_{11}x_1+a_{12}x_2=b_1, \quad \quad(17) \] \[ a_{21}x_1+a_{22}x_2=b_2, \quad \quad(18) \]

числа \(a_{ik}, b_i\), \(i,k=1,2\) считаются заданными, требуется найти неизвестные \(x_1,x_2\) . Эту систему можно решить исключением неизвестных. Например, умножим первое уравнение на \(a_{22}\) и вычтем второе, умноженное на \(a_{12}\), получим:

\[ (a_{11}a_{22}-a_{21}a_{12})x_1=b_1a_{22}-b_2a_{12}, \]

так что если \(a_{11}a_{22}-a_{21}a_{12} \neq 0, \) \[ x_1=\frac{b_1a_{22}-b_2a_{12}}{a_{11}a_{22}-a_{21}a_{12}}. \quad \quad(19) \]

Если второе уравнение умножить на \(a_{11}\) и вычесть из него первое уравнение, умноженное на \(a_{21}\), получим: \[ x_2=\frac{a_{11}b_2-a_{21}b_1}{a_{11}a_{22}-a_{21}a_{12}}. \quad \quad(20) \]

Введем следующие обозначения. Матрицей коэффициентов системы уравнений (17)-(18) назовем матрицу \[ A=\left( \begin{array}{cc} a_{11} & a_{12} \\ a_{21} & a_{22} \end{array} \right), \] столбец правых частей системы \[ B=\left (\begin{array}{c} b_1 \\b_2 \end{array} \right). \]

Тогда формулы (19), (20) можно переписать следующим образом: \[ x_1=\frac{detC_1}{detA}, x_2=\frac{detC_2}{detA}, \quad \quad(21) \] где матрица \(C_k\), \(k=1,2\), получается из матрицы \(A\) заменой ее \(k\)-того столбца на столбец \(B\). Формулы (21) называются формулами Крамера для системы из 2 уравнений с двумя неизвестными. Они описывают единственное решение системы уравнений в данном случае.

Система \(n\) уравнений, \(n\) неизвестных

Рассмотрим систему \(n\) линейных алгебраических уравнений с \(n\) неизвестными, \[ a_{11}x_1+a_{12}x_2+ .{-1}B. \]

В целом решение систем методом Крамера и методом обратной матрицы требует выполнения 2 условий: матрица коэффициентов системы должна быть квадратной ( т.е. число уравнений должно совпадать с числом неизвестных) и эта матрица должна быть невырожденной. К тому же практическая реализация этих методов связана с весьма громоздкими вычислениями, так что они имеют лишь теоретическое значение. На практике используют существенно более простой в реализации метод Гаусса, который к тому же позволяет решать и более общие системы уравнений. Этот метод описан ниже.

Решить системы методом Крамера и методом обратной матрицы.

а) \[ x_1+x_2+2x_3=-1, \] \[ 2x_1-x_2+2x_3=-4, \] \[ 4x_1+x_2+4x_3=-2. \]

б) \[ 3x_1+2x_2+x_3=5, \] \[ 2x_1+3x_2+x_3=1, \] \[ 2x_1+x_2+3x_3=11. \]

в) \[ 2x_1+x_2-x_3=2, \] \[ 3x_1+x_2-2x_3=3, \] \[ x_1+x_3=3. \]

Решение системы уравнений методом Крамера

Метод применим только в том случае, если число переменных совпадает с числом уравнений в этой системе линейных уравнений.

Необходимым условием является, чтобы определитель матрицы системы не равнялся нулю, то есть

D = det A≠0

Система из n уравнений с n неизвестными

Если определитель матрицы линейной системы не равен нулю, то система имеет единственное решение.

Решение находится по формулам:

i=0,1,2…n

D — главный определитель, составленный из числовых коэффициентов при неизвестных,

Diвспомогательный определитель, получаемый из главного заменой i -го столбца столбцом свободных членов bi.


Допустим, дана система трех линейных уравнений с тремя неизвестными, вида

главный определитель находится по формуле:

а вспомогательные по формулам:

Далее по формулам Крамера находим корни искомой системы линейных уравнений:


Пример 1

Решить систему линейных уравнений с двумя неизвестными с помощью метода Крамера

$\left\{ {\begin{array}{*{20}{c}}{2{x_1}}& + &{3{x_2}}& = &{ — 1} \\ {3{x_1}}& + &{4{x_2}}& = &{ — 1} \end{array}} \right.$

Решение

$\left\{ {\begin{array}{*{20}{c}}{2{x_1}}& + &{3{x_2}}& = &{ — 1} \\ {3{x_1}}& + &{4{x_2}}& = &{ — 1} \end{array}} \right.$

Находим определитель матрицы второго порядка системы

$\Delta  = \left| {\begin{array}{*{20}{c}}2&3 \\ 3&4 \end{array}} \right| = 8 — 9 =  — 1 \ne 0$

Имеем:

${\Delta _{\,1}} = \left| {\begin{array}{*{20}{c}} { — 1}&3 \\ { — 1}&4 \end{array}} \right|=$

$=  — 1 \cdot 4 — 3 \cdot ( — 1) =  — 1$

${\Delta _2} = \left| {\begin{array}{*{20}{c}}  2&{ — 1} \\  3&{ — 1} \end{array}} \right|=$

$= 2 \cdot ( — 1) — 3 \cdot ( — 1) = 1$

Следовательно, находим корни уравнения

${x_{\,1}} = \frac{{{\Delta _{\,1}}}}{\Delta } = \frac{{ — 1}}{{ — 1}} = 1$

${x_2} = \frac{{{\Delta _2}}}{\Delta } = \frac{1}{{ — 1}} =  — 1$


Пример 2

Решить систему линейных уравнений  с тремя неизвестными с помощью метода Крамера

$\left\{ {\begin{array}{*{20}{c}}{3{x_1} — {x_2} + {x_3} = 12} \\  {5{x_1} +{x_2} + 2{x_3} = 3} \\ {x{}_1 + {x_2} + 2{x_3} = 3} \end{array}{\text{ }}} \right.$

Решение

Найдем определитель матрицы третьего порядка, по формуле:

Определитель матрицы равен:

Определитель не равен нулю

Вычислим вспомогательные определители

Тогда получаем окончательное решение

${x_1} = \frac{{\Delta {x_1}}}{\Delta } = \frac{0}{{12}} = 0$

${x_2} = \frac{{\Delta {x_2}}}{\Delta } =  — \frac{{84}}{{12}} =  — 7$

${x_3} = \frac{{\Delta {x_3}}}{\Delta } = \frac{{60}}{{12}} = 5$

Ответ: x1=0; x2=-7; x3=5

Решение уравнений в Excel методом итераций Крамера и Гаусса

В программе Excel имеется обширный инструментарий для решения различных видов уравнений разными методами.

Рассмотрим на примерах некоторые варианты решений.

Решение уравнений методом подбора параметров Excel

Инструмент «Подбор параметра» применяется в ситуации, когда известен результат, но неизвестны аргументы. Excel подбирает значения до тех пор, пока вычисление не даст нужный итог.

Путь к команде: «Данные» – «Работа с данными» – «Анализ «что-если»» – «Подбор параметра».

Рассмотрим на примере решение квадратного уравнения х2 + 3х + 2 = 0. Порядок нахождения корня средствами Excel:

  1. Введем в ячейку В2 формулу для нахождения значения функции. В качестве аргумента применим ссылку на ячейку В1.
  2. Открываем меню инструмента «Подбор параметра». В графе «Установить в ячейку» – ссылка на ячейку В2, где находится формула. В поле «Значение» вводим 0. Это то значение, которое нужно получить. В графе «Изменяя значение ячейки» – В1. Здесь должен отобразиться отобранный параметр.
  3. После нажатия ОК отобразится результат подбора. Если нужно его сохранить, вновь нажимаем ОК. В противном случае – «Отмена».

Для подбора параметра программа использует циклический процесс. Чтобы изменить число итераций и погрешность, нужно зайти в параметры Excel. На вкладке «Формулы» установить предельное количество итераций, относительную погрешность. Поставить галочку «включить итеративные вычисления».



Как решить систему уравнений матричным методом в Excel

Дана система уравнений:

  1. Значения элементов введем в ячейки Excel в виде таблицы.
  2. Найдем обратную матрицу. Выделим диапазон, куда впоследствии будут помещены элементы матрицы (ориентируемся на количество строк и столбцов в исходной матрице). Открываем список функций (fx). В категории «Математические» находим МОБР. Аргумент – массив ячеек с элементами исходной матрицы.
  3. Нажимаем ОК – в левом верхнем углу диапазона появляется значение. Последовательно жмем кнопку F2 и сочетание клавиш Ctrl + Shift + Enter.
  4. Умножим обратную матрицу Ах-1х на матрицу В (именно в таком порядке следования множителей!). Выделяем диапазон, где впоследствии появятся элементы результирующей матрицы (ориентируемся на число строк и столбцов матрицы В). Открываем диалоговое окно математической функции МУМНОЖ. Первый диапазон – обратная матрица. Второй – матрица В.
  5. Закрываем окно с аргументами функции нажатием кнопки ОК. Последовательно нажимаем кнопку F2 и комбинацию Ctrl + Shift + Enter.

Получены корни уравнений.

Решение системы уравнений методом Крамера в Excel

Возьмем систему уравнений из предыдущего примера:

Для их решения методом Крамера вычислим определители матриц, полученных заменой одного столбца в матрице А на столбец-матрицу В.

Для расчета определителей используем функцию МОПРЕД. Аргумент – диапазон с соответствующей матрицей.

Рассчитаем также определитель матрицы А (массив – диапазон матрицы А).

Определитель системы больше 0 – решение можно найти по формуле Крамера (Dx / |A|).

Для расчета Х1: =U2/$U$1, где U2 – D1. Для расчета Х2: =U3/$U$1. И т.д. Получим корни уравнений:

Решение систем уравнений методом Гаусса в Excel

Для примера возьмем простейшую систему уравнений:

3а + 2в – 5с = -1
2а – в – 3с = 13
а + 2в – с = 9

Коэффициенты запишем в матрицу А. Свободные члены – в матрицу В.

Для наглядности свободные члены выделим заливкой. Если в первой ячейке матрицы А оказался 0, нужно поменять местами строки, чтобы здесь оказалось отличное от 0 значение.

  1. Приведем все коэффициенты при а к 0. Кроме первого уравнения. Скопируем значения в первой строке двух матриц в ячейки В6:Е6. В ячейку В7 введем формулу: =B3:Е3-$B$2:$Е$2*(B3/$B$2). Выделим диапазон В7:Е7. Нажмем F2 и сочетание клавиш Ctrl + Shift + Enter. Мы отняли от второй строки первую, умноженную на отношение первых элементов второго и первого уравнения.
  2. Копируем введенную формулу на 8 и 9 строки. Так мы избавились от коэффициентов перед а. Сохранили только первое уравнение.
  3. Приведем к 0 коэффициенты перед в в третьем и четвертом уравнении. Копируем строки 6 и 7 (только значения). Переносим их ниже, в строки 10 и 11. Эти данные должны остаться неизменными. В ячейку В12 вводим формулу массива.
  4. Прямую прогонку по методу Гаусса сделали. В обратном порядке начнем прогонять с последней строки полученной матрицы. Все элементы данной строки нужно разделить на коэффициент при с. Введем в строку формулу массива: {=B12:E12/D12}.
  5. В строке 15: отнимем от второй строки третью, умноженную на коэффициент при с второй строки ({=(B11:E11-B16:E16*D11)/C11}). В строке 14: от первой строки отнимаем вторую и третью, умноженные на соответствующие коэффициенты ({=(B10:E10-B15:E15*C10-B16:E16*D10)/B10}). В последнем столбце новой матрицы получаем корни уравнения.

Примеры решения уравнений методом итераций в Excel

Вычисления в книге должны быть настроены следующим образом:

Делается это на вкладке «Формулы» в «Параметрах Excel». Найдем корень уравнения х – х3 + 1 = 0 (а = 1, b = 2) методом итерации с применением циклических ссылок. Формула:

Хn+1 = Xn– F (Xn) / M, n = 0, 1, 2, … .

M – максимальное значение производной по модулю. Чтобы найти М, произведем вычисления:

f’ (1) = -2 * f’ (2) = -11.

Полученное значение меньше 0. Поэтому функция будет с противоположным знаком: f (х) = -х + х3 – 1. М = 11.

В ячейку А3 введем значение: а = 1. Точность – три знака после запятой. Для расчета текущего значения х в соседнюю ячейку (В3) введем формулу: =ЕСЛИ(B3=0;A3;B3-(-B3+СТЕПЕНЬ(B3;3)-1/11)).

В ячейке С3 проконтролируем значение f (x): с помощью формулы =B3-СТЕПЕНЬ(B3;3)+1.

Корень уравнения – 1,179. Введем в ячейку А3 значение 2. Получим тот же результат:

Скачать решения уравнений в Excel

Корень на заданном промежутке один.

Урок “Определители II и III порядка. Решение системы двух линейных уравнений по формуле Крамера”

Определители II и III порядка. Решение системы двух линейных уравнений по формуле Крамера.

Цель

ОБРАЗОВАТЕЛЬНЫЕ: ознакомить с определителями квадратных матриц 2-го, 3-го порядка; ознакомить с определением систем линейных уравнений и научить решать системы линейньж уравнений методом Крамера .

РАЗВИВАЮЩИЕ: развивать навыки умения вычислять определители 2-го, 3-го порядка, развивать интерес к предмету, активизировать мыслительную деятельность; ВОСПИТАТЕЛЬНЫЕ: развитие умения применять полученные знания в профессиональной деятельности;

План.

1. Определители II и III порядка.

2. Решение системы двух линейных уравнений по формуле Крамера.

Система двух линейных уравнений с двумя переменными в общем виде имеет вид qx+blY=q а2Х + Ду =

Рёшением системы уравнений называется упорядоченная пара чисел, удовлетворяющая каждому уравнению этой системы. При решении такой системы могут быть использованы известные методы: 1) подстановки; 2) алгебраического сложения; З) графически.

Но существует ещё метод решения, который особенно удобен в том случае, когда коэффициенты q ; а2 ; ; b2 отличны от единицы или содержат буквенные выражения.

Имеем систему уравнений qx + Ду = q

                  Число аф2 —                            называется определителем второго порядка.

Вертикальные прямые знак определителя. Обозначается определитель знаком ” Л ” (дельта).

Итак определитель — это число, которое вычисляется по определенному правилу

Х = аф2 —

— первый столбик (коэффициенты при переменной х)

— второй столбик (коэффициенты при переменной у)

 bl — первая при переменных первого уравнения) а b2 — вторая строчка (коэффициенты при переменных второго,уравнения)

Определители при переменных дх и д получаются из определителя системы путем замены соответствующего столбика столбиком из свободных членов.

                                   = сф2 — сф                                                           = alC2 — а2С1

Для нахождения значений переменных х и у используются формулы х =

дд которые называются формулами Крамера.

Исследуем

1)     Если д 0 — система имеет единственное решение

2)     Если Л = 0, но Ах 0 или д     система не имеет решения

З) Если д = 0 и д = 0 и д = 0 — система имеет множество решений.

= —39

Ответ: (1; 1). —39

39

1.     Определитель не изменится, если его строки заменить столбцами, и наоборот

2.     При перестановке двух столбцов (строчек) определитель меняет свой знак на противоположный.

З. Определитель, имеющий два одинаковых или пропорциональных столбца (строчки)

равен нулю.

4. Общий множитель столбца (строчки) определителя можно вьшести за знак определителя.

Решить систему уравнений:

29 д 0, система имеет единственное решение

2-19–20-38= —58 Ответ: (1; —2).

        Д       29                        д       -58

         Д      29                         Д       29

При решении системы двух линейных уравнений с двумя переменньпии следует помнить, что решение можно выполнить любым из известных методов решения, просто следует выбрать каким методом более рационально для данной системы.

3х 8У = 31

3)

Решим систему всеми способами, т.е. убедимся, что результат получается одинаковый и определимся, какой из методов более рационально применим для данной системы. 1) Способ подстановки.

   8 у = 31           Зх + 8у = 31

310 80

Решаем второе уравнение относительно

                                                                                                                           З          З

знаменателю и так как З 0 ,то

310-80y+21y=15

-59у=15-ЗIО

-295

-59у=-295; у-Ответ: х

—59

                                   31 8           31 40         9

У = 5, тогда х

2) Способ алгебраического сложения


—5, приведем к общему


3х+8у=З1 уравняем по модулю коэффициенты при х, для этого умножим первое -10х-7у=_5 уравнение на 10, а второе — на З.

        3х+8у=З1        10               30х+8Оу=З1О

почленно сложим

-10х-7у-30х-20=-15

59у = 295

подставим у= 5 в тобое из уравнений системы, например в первое, и найдем х 3х+86=31

3х+40=31

3х = —9

получаем х —З; у 5, как и в первом случае.

З) графически (следует помнить, что результаты могут быть получены приближенно, что можно объяснить нашим зрением, умением проводить линии, выбором масштаба, неудобством записи числа и т.д.)

       + 8у = 31              графиком каждого уравнения является прямая, а прямая определяется

-10х-7у=-5 двумя точками.

3х 8У = 31

                        31       7

                         8        8

          31-6 25      1

               8         8        8

-10х-7у=-5

10х+7у=5

5

4) С помощью определителя:

3х+8у=З1

-10х-7у=-5

3х+8у=З1

=21-80— —59 0 единственное решение 10х+7у=5

31 8з 31 дд 15-310=-295

5 710 5

       177                                                       -295

-3;

—59               —59 Ответ: (—3; 5).

Каким же способом более рационально мо-жно было решить эту систему? Вы правы, конечно с помощью определителя.

Самостоятельно (любьлт способом)

   4х 95, = 22а          2х-Зу=11

1)                                  2)                                3)

       11х+5у = а                  6х—9у = 33

Система трех линейных уравнений с тремя переменньми имеет вид:

На I курсе рассматривается решение такой системы с помощью определителя третьего порядка.

Выражение, составленное из коэффициентов при переменных в виде таблицы

называется определителем третьего поряда.

Определитель третьего порядка вычислить можно через определитель второго порядка или по правилу Саррюса (правило треугольника).

1)         Через определитель II порядка.

а22

32

           22 33               32 23                                                                                                                  —а м а22

Выделяем аи и мысленно вычеркиваем по столбику и строчке, оставшиеся члены составляют определитель второго порядка. Берем а] 2 с противоположньпи знаком и вычеркиваем первую строчку и второй столбик, оставшиеся члены составляют определитель II порядка. Аналогично берем ап и вычеркиваем первую строчку и трети.й столбик, оставшиеся члены составляют определитель II порядка.

Выполняем вычисления определителей II порядка по известному уже нам правилу. Например:

—4

+2.

= -66-22+132 -88+132

2) Правило треугольника (Саррюса). Рассмотрим схематически

а) (основания равнобедренных треугольников параллельны главной диагонали)

Ь) (основания равнобедренных треугольников параллельны побочной диагонали)

Пример: (возьмем тот же определитель)

—96 +140 = 44

В дальнейшем запись будем вести так

Определители дх;д ; д: получаются из определителя системы путем замены

соответствующего столбика столбиком из свободных членов и вычисляются по тому же правилу, что и определитель системы.

Для нахождения значений х; у; г пользуются правилами Крамера д: д

Исследование:

1.     Если д 0, то система имеет единственное решение

2.     Если д = 0, то система несовместима, т.е. не имеет решения

(либо дх 0; либо ду 0; либо д: 0)

3.     Если д = 0, то система неопределенна, т.е. имеет множество решений       

Определитель III порядка обладает всеми свойствами определителя II порядка.

Например, решить систему уравнений

так как коэффициенты при переменных и свободные члены пропорциональны.

4 – 2 – 2 8 2      6 – з – з – 12 з

Система имеет множество решений, т.е. неопределена.

+119 = 55 0

д и д: можно уже не находить. Следовательно система не имеет решения.

Самостоятельно:

(А = —37; х = 1; у = 3;z 4)

(д () система не имеет решения

( д = о;дгд2 = О система имеет множество решений)

Контрольные вопросы:

1.      Что такое системы уравнений II (III) порядка?

2.      Что называют элементами определителя?

З. По какой формуле находится определитель системы?

как методом крамера решить систему уравнений

Вы искали как методом крамера решить систему уравнений? На нашем сайте вы можете получить ответ на любой математический вопрос здесь. Подробное решение с описанием и пояснениями поможет вам разобраться даже с самой сложной задачей и как найти дискриминант матрицы по методу крамера, не исключение. Мы поможем вам подготовиться к домашним работам, контрольным, олимпиадам, а так же к поступлению в вуз. И какой бы пример, какой бы запрос по математике вы не ввели – у нас уже есть решение. Например, «как методом крамера решить систему уравнений».

Применение различных математических задач, калькуляторов, уравнений и функций широко распространено в нашей жизни. Они используются во многих расчетах, строительстве сооружений и даже спорте. Математику человек использовал еще в древности и с тех пор их применение только возрастает. Однако сейчас наука не стоит на месте и мы можем наслаждаться плодами ее деятельности, такими, например, как онлайн-калькулятор, который может решить задачи, такие, как как методом крамера решить систему уравнений,как найти дискриминант матрицы по методу крамера,как решать линейные уравнения методом крамера,как решать матрицу методом крамера,как решать матрицы методом крамера,как решать метод крамера,как решать методом крамера,как решать методом крамера линейные уравнения,как решать методом крамера матрицы,как решать систему уравнений методом крамера,как решить матрицу методом крамера,как решить методом крамера систему,как решить методом крамера систему уравнений,как решить систему линейных уравнений методом крамера,как решить систему методом крамера,как решить систему уравнений методом крамера,крамер матрица,крамер метод,крамер формулы,крамера,крамера матрица,крамера метод пример,крамера метод это,линейные уравнения методом крамера,матрица крамер,матрица крамера,матрица метод крамера,матрица методом крамера,матрицу решить методом крамера,матрицы метод крамера,матрицы метод крамера примеры,матрицы примеры метод крамера,матрицы теорема крамера,метод гаусса и крамера,метод гаусса и метод крамера,метод гаусса крамера и,метод гаусса крамера и матричный метод,метод гаусса метод крамера матричный метод,метод крамер,метод крамера,метод крамера 4 на 4,метод крамера 4х4,метод крамера гаусса и,метод крамера для матрицы 4 порядка,метод крамера для решения систем линейных уравнений,метод крамера для чайников,метод крамера и гаусса,метод крамера и матричный метод,метод крамера и метод гаусса,метод крамера и метод гаусса решения систем линейных уравнений,метод крамера как решать,метод крамера матрица,метод крамера матрицы,метод крамера матрицы примеры,метод крамера метод гаусса,метод крамера метод гаусса и,метод крамера метод гаусса матричный метод,метод крамера пример,метод крамера примеры,метод крамера примеры с решением,метод крамера решение,метод крамера решение матриц,метод крамера решение систем линейных уравнений,метод крамера решения,метод крамера решения систем линейных уравнений,метод крамера система линейных уравнений,метод крамера системы линейных уравнений,метод крамера слау,метод крамера теория,метод крамера формула,метод крамера формулы,метод крамера это,метод обратной матрицы метод крамера,метод решение крамера,метод решения крамера,метод слау крамера,метода крамера,методом крамера,методом крамера как решать,методом крамера матрица,методом крамера решить,методом крамера решить матрицу,методом крамера решить системы уравнений,методом крамера решить уравнение,по крамеру решение,по формулам крамера,по формулам крамера решить систему,по формулам крамера решить систему линейных уравнений,по формулам крамера решить систему уравнений,по формуле крамера решить систему,по формуле крамера решить систему линейных уравнений,по формуле крамера решить систему уравнений,правила крамера,правило крамера,правило крамера решения систем,правило крамера решения систем линейных уравнений,пример метод крамера,примеры линейных уравнений решение методом крамера,примеры метод крамера,примеры решение линейных уравнений методом крамера,примеры формула крамера,решение линейных систем уравнений по формулам крамера,решение линейных уравнений методом крамера,решение линейных уравнений методом крамера примеры,решение матриц метод крамера,решение матриц методом крамера,решение матриц по методу крамера,решение матрицы методом крамера,решение метод крамера,решение методом крамера,решение по крамеру,решение по формуле крамера,решение систем линейных уравнений метод крамера,решение систем линейных уравнений методом крамера,решение систем линейных уравнений методом крамера методом гаусса,решение систем линейных уравнений по формулам крамера,решение систем методом крамера,решение систем по формулам крамера,решение систем уравнений методом крамера,решение систем уравнений методом крамера примеры с решением,решение систем уравнений по формулам крамера,решение системных уравнений методом крамера,решение системы линейных уравнений методом крамера,решение системы методом крамера,решение системы по формулам крамера,решение системы уравнений методом крамера,решение слау методом крамера,решение уравнений методом крамера,решение уравнений по формулам крамера,решение уравнения методом крамера,решения метод крамера,решите систему линейных уравнений методом крамера,решите систему уравнений методом крамера,решите систему уравнений по формулам крамера,решить матрицу методом крамера,решить методом крамера,решить методом крамера системы уравнений,решить методом крамера слау,решить методом крамера уравнение,решить по правилу крамера систему,решить по правилу крамера систему уравнений,решить по формулам крамера систему,решить по формулам крамера систему уравнений,решить по формуле крамера систему,решить по формуле крамера систему уравнений,решить систему алгебраических линейных уравнений методом крамера,решить систему линейных уравнений методом крамера,решить систему линейных уравнений по формулам крамера,решить систему линейных уравнений по формуле крамера,решить систему методом гаусса и методом крамера,решить систему методом крамера,решить систему методом крамера и методом гаусса,решить систему по правилу крамера,решить систему по формулам крамера,решить систему по формуле крамера,решить систему уравнений методом крамера,решить систему уравнений по правилу крамера,решить систему уравнений по формулам крамера,решить систему уравнений по формуле крамера,решить системы уравнений методом крамера,решить слау методом крамера,решить уравнение методом крамера,система крамера,система линейных уравнений метод крамера,система линейных уравнений методом крамера,система уравнений методом крамера,систему линейных уравнений решить по формулам крамера,систему уравнений решить по правилу крамера,системы линейных уравнений метод крамера,слау метод крамера,слау методом крамера,способ крамера,теорема крамера матрицы,теория крамера,теория метод крамера,уравнение крамера,уравнение методом крамера,формула крамера,формула крамера для решения,формула крамера для решения системы,формула крамера для решения системы линейных уравнений,формула крамера примеры,формула метод крамера,формулам крамера,формулы крамер,формулы крамера,формулы крамера для решения систем,формулы крамера для решения систем линейных уравнений,формулы метод крамера. На этой странице вы найдёте калькулятор, который поможет решить любой вопрос, в том числе и как методом крамера решить систему уравнений. Просто введите задачу в окошко и нажмите «решить» здесь (например, как решать линейные уравнения методом крамера).

Где можно решить любую задачу по математике, а так же как методом крамера решить систему уравнений Онлайн?

Решить задачу как методом крамера решить систему уравнений вы можете на нашем сайте https://pocketteacher.ru. Бесплатный онлайн решатель позволит решить онлайн задачу любой сложности за считанные секунды. Все, что вам необходимо сделать – это просто ввести свои данные в решателе. Так же вы можете посмотреть видео инструкцию и узнать, как правильно ввести вашу задачу на нашем сайте. А если у вас остались вопросы, то вы можете задать их в чате снизу слева на странице калькулятора.

9.8: Решение систем с помощью правила Крамера

Мы узнали, как решать системы уравнений с двумя переменными и тремя переменными, а также с помощью нескольких методов: подстановки, сложения, исключения Гаусса, использования обратной матрицы и построения графиков. Некоторые из этих методов применять проще, чем другие, и они более подходят в определенных ситуациях. В этом разделе мы изучим еще две стратегии решения систем уравнений.

Вычисление определителя матрицы 2 × 2

Определитель – это действительное число, которое может быть очень полезно в математике, потому что у него есть несколько приложений, таких как вычисление площади, объема и других величин.Здесь мы будем использовать определители, чтобы определить, является ли матрица обратимой, используя элементы квадратной матрицы, чтобы определить, существует ли решение системы уравнений. Однако, возможно, одним из наиболее интересных приложений является их использование в криптографии. Защищенные сигналы или сообщения иногда отправляются в виде матрицы. Расшифровать данные можно только с помощью обратимой матрицы и определителя. В наших целях мы ориентируемся на определитель как на показатель обратимости матрицы.Вычисление определителя матрицы требует следования определенным шаблонам, описанным в этом разделе.

НАЙТИ ОПРЕДЕЛЕНИЕ МАТРИЦЫ 2 × 2

Определитель матрицы 2 × 2, учитывая

\ (A = \ begin {bmatrix} a & b \\ c & d \ end {bmatrix} \)

определяется как

Обратите внимание на изменение обозначений. Есть несколько способов указать определитель, включая \ (\ det (A) \) и замену скобок в матрице прямыми линиями, \ (| A | \).

Пример \ (\ PageIndex {1} \): поиск определителя матрицы \ (2 × 2 \)

Найдите определитель заданной матрицы.

\ (A = \ begin {bmatrix} 5 & 2 \\ – 6 & 3 \ end {bmatrix} \)

Решение

\ [\ begin {align *} \ det (A) & = \ begin {vmatrix} 5 & 2 \\ – 6 & 3 \ end {vmatrix} \\ & = 5 (3) – (- 6) (2) \\ & = 27 \ end {align *} \]

Использование правила Крамера для решения системы двух уравнений с двумя переменными

Теперь мы представим последний метод решения систем уравнений, использующий определители.Этот метод, известный как правило Крамера , восходит к середине 18 века и назван в честь своего новатора, швейцарского математика Габриэля Крамера (1704-1752), который представил его в 1750 году в Introduction à l’Analyse des lignes Courbes algébriques . Правило Крамера – это жизнеспособный и эффективный метод поиска решений систем с произвольным числом неизвестных при условии, что у нас есть такое же количество уравнений, что и неизвестных.

Правило Крамера даст нам единственное решение системы уравнений, если оно существует.Однако, если система не имеет решения или бесконечное количество решений, это будет обозначено нулевым определителем. Чтобы выяснить, является ли система непоследовательной или зависимой, необходимо использовать другой метод, например исключение.

Чтобы понять правило Крамера, давайте внимательно рассмотрим, как мы решаем системы линейных уравнений с использованием основных операций со строками. Рассмотрим систему двух уравнений с двумя переменными.

\ [\ begin {align} a_1x + b_1y & = c_1 (1) \ label {eq1} \\ a_2x + b_2y & = c_2 (2) \ label {eq2} \\ \ end {align} \]

Мы исключаем одну переменную, используя операции со строками, и решаем для другой.Скажите, что мы хотим найти \ (x \). Если уравнение \ ref {eq2} умножается на коэффициент, противоположный коэффициенту \ (y \) в уравнении \ ref {eq1}, уравнение \ ref {eq1} умножается на коэффициент при \ (y \) в уравнении \ ref {eq2}, и мы добавляем два уравнения, переменная \ (y \) будет удалена.

\ [\ begin {align *} & b_2a_1x + b_2b_1y = b_2c_1 & \ text {Multiply} R_1 \ text {by} b_2 \\ – & \ underline {b_1a_2x − b_1b_2y = −b_1c_2} & \ text {Multiply} R_2 \ text {by} −b_1 \\ & b_2a_1x − b_1a_2x = b_2c_1 − b_1c_2 \ end {align *} \]

Теперь решите относительно \ (x \).

\ [\ begin {align *} b_2a_1x − b_1a_2x & = b_2c_1 − b_1c_2 \\ x (b_2a_1 − b_1a_2) & = b_2c_1 − b_1c_2 \\ x & = \ dfrac {b_2c_1 − b_1c_2} {b_2a_1 − b_1a_2} = \ dfrac {\ begin {bmatrix} c_1 & b_1 \\ c_2 & b_2 \ end {bmatrix}} {\ begin {bmatrix} a_1 & b_1 \\ a_2 & b_2 \ end {bmatrix}} \ end {align *} \]

Аналогичным образом, чтобы найти \ (y \), мы исключим \ (x \).

\ [\ begin {align *} & a_2a_1x + a_2b_1y = a_2c_1 & \ text {Multiply} R_1 \ text {by} a_2 \\ – & \ underline {a_1a_2x − a_1b_2y = −a_1c_2} & \ text {Multiply} R_2 \ текст {by} −a_1 \\ & a_2b_1y − a_1b_2y = a_2c_1 − a_1c_2 \ end {align *} \]

Решение относительно \ (y \) дает

\ [\ begin {align *} a_2b_1y − a_1b_2y & = a_2c_1 − a_1c_2 \\ y (a_2b_1 − a_1b_2) & = a_2c_1 − a_1c_2 \\ y & = \ dfrac {a_2c_1 − a_1c_2} {a_2b_1 − a_1b_2} = \ dfrac {a_1c_2 − a_2c_1} {a_1b_2 − a_2b_1} = \ dfrac {\ begin {bmatrix} a_1 & c_1 \\ a_2 & c_2 \ end {bmatrix}} {\ begin {bmatrix} a_1 & b_1 \\ a_2 & b_2 \ end {bmatrix}} \ end {align * } \]

Обратите внимание, что знаменатель для \ (x \) и \ (y \) является определителем матрицы коэффициентов.

Мы можем использовать эти формулы для решения относительно \ (x \) и \ (y \), но правило Крамера также вводит новые обозначения:

  • \ (D \): определитель матрицы коэффициентов
  • \ (D_x \): определитель числителя в решении \ (x \)

    \ [x = \ dfrac {D_x} {D} \]

  • \ (D_y \): определитель числителя в решении \ (y \)

    \ [y = \ dfrac {D_y} {D} \]

Ключ к правилу Крамера заключается в замене интересующего столбца переменных столбцом констант и вычислении детерминантов.Тогда мы можем выразить \ (x \) и \ (y \) как частное двух определителей.

ПРАВИЛО КРЕМЕРА ДЛЯ СИСТЕМ \ (2 × 2 \)

Правило Крамера – это метод, использующий детерминанты для решения систем уравнений, которые имеют то же количество уравнений, что и переменные.

Рассмотрим систему двух линейных уравнений с двумя переменными.

\ [\ begin {align *} a_1x + b_1y & = c_1 \\ a_2x + b_2y & = c_2 \ end {align *} \]

Решение, использующее правило Крамера, дается как

\ [\ begin {align} x & = \ dfrac {D_x} {D} = \ dfrac {\ begin {bmatrix} c_1 & b_1 \\ c_2 & b_2 \ end {bmatrix}} {\ begin {bmatrix} a_1 & b_1 \\ a_2 & b_2 \ end { bmatrix}} \; , D \ neq 0 \\ y & = \ dfrac {D_y} {D} = \ dfrac {\ begin {bmatrix} a_1 & c_1 \\ a_2 & c_2 \ end {bmatrix}} {\ begin {bmatrix} a_1 & b_1 \\ a_2 & b_2 \ end {bmatrix }} \; , D \ neq 0 \ end {align} \]

Если мы решаем для \ (x \), столбец \ (x \) заменяется постоянным столбцом.Если мы решаем для \ (y \), столбец \ (y \) заменяется постоянным столбцом.

Пример \ (\ PageIndex {2} \): использование правила Крамера для решения системы \ (2 × 2 \)

Решите следующую систему \ (2 × 2 \), используя правило Крамера.

\ [\ begin {align *} 12x + 3y & = 15 \\ 2x-3y & = 13 \ end {align *} \]

Решение

Решите относительно \ (x \).

\ [\ begin {align *} x & = \ dfrac {D_x} {D} \\ & = \ dfrac {\ begin {bmatrix} 15 & 3 \\ 13 & -3 \ end {bmatrix}} {\ begin {bmatrix} 12 & 3 \\ 2 & -3 \ end {bmatrix}} \\ & = \ dfrac {-45-39} {- 36-6} \\ & = \ dfrac {-84} {- 42} \\ & = 2 \ end {align *} \]

Решите относительно \ (y \).

\ [\ begin {align *} y & = \ dfrac {D_y} {D} \\ & = \ dfrac {\ begin {bmatrix} 12 & 15 \\ 2 & 13 \ end {bmatrix}} {\ begin {bmatrix} 12 & 3 \\ 2 & -3 \ end {bmatrix}} \\ & = \ dfrac {156-30} {- 36-6} \\ & = – \ dfrac {126} {42} \\ & = -3 \ end {align * } \]

Решение: \ ((2, −3) \).

Упражнение \ (\ PageIndex {1} \)

Используйте правило Крамера для решения системы уравнений \ (2 × 2 \).

\ [\ begin {align *} x + 2y & = -11 \\ -2x + y & = -13 \ end {align *} \]

Ответ

\ ((3, −7) \)

Вычисление определителя матрицы 3 × 3

Найти определитель матрицы 2 × 2 несложно, но найти определитель матрицы 3 × 3 сложнее.Один из способов – увеличить матрицу 3 × 3 повторением первых двух столбцов, получив матрицу 3 × 5. Затем мы вычисляем сумму произведений записей на по каждой из трех диагоналей (от верхнего левого угла к нижнему правому) и вычитаем произведения записей на по каждой из трех диагоналей (нижний левый верхний правый). Это легче понять с помощью наглядного пособия и примера.

Найдите определитель матрицы 3 × 3.

\ (A = \ begin {bmatrix} a_1 & b_1 & c_1 \\ a_2 & b_2 & c_2 \\ a_3 & b_3 & c_3 \ end {bmatrix} \)

  1. Дополнение \ (A \) с первыми двумя столбцами.

    \ (\ det (A) = \ left | \ begin {array} {ccc | cc} a_1 & b_1 & c_1 & a_1 & b_1 \\ a_2 & b_2 & c_2 & a_2 & b_2 \\ a_3 & b_3 & c_3 & a_3 & b_3 \ end {array} \ right | \)

  2. С верхнего левого угла в нижний правый: умножение значений по первой диагонали. Добавьте результат к произведению записей по второй диагонали. Добавьте этот результат к произведению записей по третьей диагонали.
  3. От левого нижнего угла до правого верхнего: вычтите произведение входов вверх по первой диагонали.Из этого результата вычтите произведение входов вверх по второй диагонали. Из этого результата вычтите произведение входов до третьей диагонали.

Алгебра выглядит следующим образом:

\ (| A | = a_1b_2c_3 + b_1c_2a_3 + c_1a_2b_3 − a_3b_2c_1 − b_3c_2a_1 − c_3a_2b_1 \)

Пример \ (\ PageIndex {3} \): поиск определителя матрицы 3 × 3

Найдите определитель матрицы \ (3 × 3 \) при

\ (A = \ begin {bmatrix} 0 & 2 & 1 \\ 3 & −1 & 1 \\ 4 & 0 & 1 \ end {bmatrix} \)

Решение

Дополните матрицу первыми двумя столбцами, а затем следуйте формуле.Таким образом,

\ [\ begin {align *} | А | & = \ left | \ begin {array} {ccc | cc} 0 & 2 & 1 & 0 & 2 \\ 3 & -1 & 1 & 3 & -1 \\ 4 & 0 & 1 & 4 & 0 \ end {array} \ right | \\ & = 0 (−1) (1) +2 (1) (4) +1 (3) (0) −4 (−1) (1) −0 (1) (0) −1 (3) (2) \\ & = 0 + 8 + 0 + 4−0−6 \\ & = 6 \ end {align *} \]

Упражнение \ (\ PageIndex {2} \)

Найдите определитель матрицы 3 × 3.

\ (\ det (A) = \ begin {vmatrix} 1 & −3 & 7 \\ 1 & 1 & 1 \\ 1 & −2 & 3 \ end {vmatrix} \)

Ответ

\ (- 10 \)

Q&A: Можем ли мы использовать тот же метод, чтобы найти определитель большей матрицы?

Нет, этот метод работает только для матриц 2 × 2 и 3 × 3.Для больших матриц лучше всего использовать графическую утилиту или компьютерное программное обеспечение.

Использование правила Крамера для решения системы трех уравнений с тремя переменными

Теперь, когда мы можем найти определитель матрицы \ (3 × 3 \), мы можем применить правило Крамера для решения системы трех уравнений с тремя переменными. Правило Крамера простое и следует шаблону, соответствующему правилу Крамера для матриц \ (2 × 2 \). Однако по мере увеличения порядка матрицы до \ (3 × 3 \) требуется гораздо больше вычислений.

Когда мы вычисляем, что определитель равен нулю, правило Крамера не дает никаких указаний на то, что у системы нет решения или есть бесконечное количество решений. Чтобы выяснить это, мы должны выполнить устранение в системе.

Рассмотрим систему уравнений \ (3 × 3 \).

\ [\ begin {align} a_1x + b_1y + c_1z & = \ color {blue} d_1 \\ a_2x + b_2y + c_2z & = \ color {blue} d_2 \\ a_3x + b_3y + c_3z & = \ color {blue} d_3 \\ \ end {align} \]

\ (x = \ dfrac {D_x} {D} \), \ (y = \ dfrac {D_y} {D} \), \ (z = \ dfrac {D_z} {D} \), \ (D ≠ 0 \)

где

\ [D = \ begin {vmatrix} a_1 & b_1 & c_1 \\ a_2 & b_2 & c_2 \\ a_3 & b_3 & c_3 \ end {vmatrix} \; , \; D_x = \ begin {vmatrix} \ color {blue} d_1 & b_1 & c_1 \\ \ color {blue} d_2 & b_2 & c_2 \\ \ color {blue} d_3 & b_3 & c_3 \ end {vmatrix} \; , \; D_y = \ begin {vmatrix} a_1 & \ color {blue} d_1 & c_1 \\ a_2 & \ color {blue} d_2 & c_2 \\ a_3 & \ color {blue} d_3 & c_3 \ end {vmatrix} \; , \; D_z = \ begin {vmatrix} a_1 & b_1 & \ color {blue} d_1 \\ a_2 & b_2 & \ color {blue} d_2 \\ a_3 & b_3 & \ color {blue} d_3 \ end {vmatrix} \]

Если мы записываем определитель \ (D_x \), мы заменяем столбец \ (x \) постоянным столбцом.Если мы пишем определитель \ (D_y \), мы заменяем столбец y на столбец констант. Если мы пишем определитель \ (D_z \), мы заменяем столбец \ (z \) постоянным столбцом. Всегда проверяйте ответ.

Пример \ (\ PageIndex {4} \): решение системы \ (3 × 3 \) с использованием правила Крамера

Найдите решение данной системы \ (3 × 3 \), используя правило Крамера.

\ [\ begin {align *} x + y-z & = 6 \\ 3x-2y + z & = -5 \\ x + 3y-2z & = 14 \ end {align *} \]

Решение

Используйте правило Крамера.

\ (D = \ begin {vmatrix} 1 & 1 & −1 \\ 3 & −2 & 1 \\ 1 & 3 & −2 \ end {vmatrix} \), \ (D_x = \ begin {vmatrix} 6 & 1 & −1 \\ – 5 & −2 & 1 \ \ 14 & 3 & −2 \ end {vmatrix} \), \ (D_y = \ begin {vmatrix} 1 & 6 & −1 \\ 3 & −5 & 1 \\ 1 & 14 & −2 \ end {vmatrix} \), \ (D_z = \ begin {vmatrix } 1 & 1 & 6 \\ 3 & −2 & −5 \\ ​​1 & 3 & 14 \ end {vmatrix} \)

Затем,

\ [\ begin {align *} x & = \ dfrac {D_x} {D} & = \ dfrac {-3} {- 3} & = 1 \\ y & = \ dfrac {D_y} {D} & = \ dfrac {-9} {- 3} & = 3 \\ z & = \ dfrac {D_z} {D} & = \ dfrac {6} {- 3} & = -2 \\ \ end {align *} \]

Решение: \ ((1,3, −2) \).

Упражнение \ (\ PageIndex {3} \)

Используйте правило Крамера, чтобы решить матрицу \ (3 × 3 \).

\ [\ begin {align *} x-3y + 7z & = 13 \\ x + y + z & = 1 \\ x-2y + 3z & = 4 \ end {align *} \]

Ответ

\ (\ left (−2, \ dfrac {3} {5}, \ dfrac {12} {5} \ right) \)

Пример \ (\ PageIndex {5A} \): Использование правила Крамера для решения несовместимой системы

Решите систему уравнений, используя правило Крамера.

\ [\ begin {align} 3x-2y & = 4 \ label {eq3} \\ 6x-4y & = 0 \ label {eq4} \ end {align} \]

Решение

Начнем с нахождения определителей \ (D \), \ (D_x \) и \ (D_y \).

\ (D = \ begin {vmatrix} 3 & −2 \\ 6 & −4 \ end {vmatrix} = 3 (−4) −6 (−2) = 0 \)

Мы знаем, что нулевой определитель означает, что либо система не имеет решения, либо имеет бесконечное количество решений. Чтобы узнать, какой из них, мы используем процесс исключения. Наша цель – исключить одну из переменных.

  1. Умножьте уравнение \ ref {eq3} на \ (- 2 \).
  2. Добавьте результат в уравнение \ ref {eq4}.

\ [\ begin {align *} & −6x + 4y = −8 \\ & \; \; \; \ underline {6x − 4y = 0} \\ & \; \; \; \; \; \ ; \; \; \; \; 0 = −8 \ end {align *} \]

Получаем уравнение \ (0 = −8 \), которое неверно. Следовательно, у системы нет решения. График системы показывает две параллельные линии. См. Рисунок \ (\ PageIndex {1} \).

Рисунок \ (\ PageIndex {1} \)

Пример \ (\ PageIndex {5B} \): использование правила Крамера для решения зависимой системы

Решите систему с бесконечным количеством решений.

\ [\ begin {align} x-2y + 3z & = 0 \ label {eq5} \\ 3x + y-2z & = 0 \ label {eq6} \\ 2x-4y + 6z & = 0 \ label {eq7} \ end {align} \]

Решение

Давайте сначала найдем определитель. Создайте матрицу, дополненную первыми двумя столбцами.

\ (\ left | \ begin {array} {ccc | cc} 1 & −2 & 3 & 1 & -2 \\ 3 & 1 & −2 & 3 & 1 \\ 2 & −4 & 6 & 2 & -4 \ end {array} \ right | \)

Затем,

\ (1 (1) (6) + (- 2) (- 2) (2) +3 (3) (- 4) −2 (1) (3) – (- 4) (- 2) (1 ) −6 (3) (- 2) = 0 \)

Поскольку определитель равен нулю, решения либо нет, либо существует бесконечное количество решений.Чтобы выяснить это, нам нужно провести отбор.

1. Умножьте уравнение \ ref {eq5} на \ (- 2 \) и добавьте результат к уравнению \ ref {eq7}:

\ [\ begin {align *} & −2x + 4y − 6x = 0 \\ & \; \; \ underline {2x − 4y + 6z = 0} \\ & \; \; \; \; \; \ ; \; \; \; \; \; \; \; \; \; \; 0 = 0 \ end {align *} \]

2. Получение ответа \ (0 = 0 \), утверждение, которое всегда верно, означает, что система имеет бесконечное количество решений. Изобразив систему, мы можем увидеть, что две плоскости одинаковы, и обе они пересекают третью плоскость по прямой.См. Рисунок \ (\ PageIndex {2} \).

Рисунок \ (\ PageIndex {2} \)

Понимание свойств детерминантов

Есть много свойств определителей. Здесь перечислены некоторые свойства, которые могут быть полезны при вычислении определителя матрицы.

СВОЙСТВА ДЕТЕРМИНАНТОВ

  1. Если матрица имеет верхнюю треугольную форму, определитель равен произведению элементов по главной диагонали.
  2. Когда две строки меняются местами, определитель меняет знак.{−1} \) – величина, обратная определителю матрицы \ (A \).
  3. Если какая-либо строка или столбец умножается на константу, определитель умножается на тот же коэффициент.

Пример \ (\ PageIndex {6} \): иллюстрация свойств детерминантов

Проиллюстрируйте каждое из свойств определителей.

Решение

Свойство 1 утверждает, что если матрица имеет верхнюю треугольную форму, определитель является произведением элементов по главной диагонали.

\ (A = \ begin {bmatrix} 1 & 2 & 3 \\ 0 & 2 & 1 \\ 0 & 0 & −1 \ end {bmatrix} \)

Дополните \ (A \) первыми двумя столбцами.

\ (A = \ left [\ begin {array} {ccc | cc} 1 & 2 & 3 & 1 & 2 \\ 0 & 2 & 1 & 0 & 2 \\ 0 & 0 & −1 & 0 & 0 \ end {array} \ right] \)

Затем

\ [\ begin {align *} \ det (A) & = 1 (2) (- 1) +2 (1) (0) +3 (0) (0) -0 (2) (3) -0 (1) (1) +1 (0) (2) \\ & = -2 \ end {align *} \]

Свойство 2 утверждает, что перестановка строк меняет знак.Учитывая

\ [\ begin {align *} B & = \ begin {bmatrix} 4 & -3 \\ – 1 & 5 \ end {bmatrix} \\ \ det (B) & = (4) (5) – (- 1) (- 3) \\ & = 20-3 \\ & = 17 \ end {align *} \]

Свойство 3 утверждает, что если две строки или два столбца идентичны, определитель равен нулю.

\ [\ begin {align *} A & = \ left [\ begin {array} {ccc | cc} 1 & 2 & 2 & 1 & 2 \\ 2 & 2 & 2 & 2 & 2 \\ – 1 & 2 & 2 & -1 & 2 \ end {array} \ right] \\ \ det (A) & = 1 (2) (2) +2 (2) (- 1) +2 (2) (2) +1 (2) (2) -2 (2) (1) -2 (2) (2) \ \ & = 4-4 + 8 + 4-4-8 \\ & = 0 \ end {align *} \]

Свойство 4 утверждает, что если строка или столбец равны нулю, определитель равен нулю.{-1}) & = – 2 \ left (- \ dfrac {1} {2} \ right) – \ dfrac {3} {2} (1) \\ & = – \ dfrac {1} {2} \ конец {выравнивание *} \]

Свойство 6 утверждает, что если любая строка или столбец матрицы умножается на константу, определитель умножается на тот же коэффициент. Таким образом,

Пример \ (\ PageIndex {7} \): использование правила Крамера и определяющих свойств для решения системы

Найдите решение данной системы \ (3 × 3 \).

Решение

Используя правило Крамера, имеем

\ (D = \ begin {bmatrix} 2 & 4 & 4 \\ 3 & 7 & 7 \\ 1 & 2 & 2 \ end {bmatrix} \)

Обратите внимание, что второй и третий столбцы идентичны.Согласно свойству 3 определитель будет равен нулю, поэтому решения либо нет, либо существует бесконечное число решений. Чтобы выяснить это, нам нужно провести отбор.

1. Умножьте уравнение \ ref {eq10} на \ (- 2 \) и добавьте результат в уравнение \ ref {eq8}.

Получение противоречивого утверждения означает, что система не имеет решения.

Медиа

Получите доступ к этим онлайн-ресурсам для получения дополнительных инструкций и практики с правилом Крамера.

Использование правила Крамера для решения системы двух уравнений с двумя переменными

Вычисление определителя матрицы 2 × 2

Определитель – это действительное число, которое может быть очень полезно в математике, потому что у него есть несколько приложений, таких как вычисление площади, объема и других величин.Здесь мы будем использовать определители, чтобы определить, является ли матрица обратимой, используя элементы квадратной матрицы , чтобы определить, существует ли решение системы уравнений. Однако, возможно, одним из наиболее интересных приложений является их использование в криптографии. Защищенные сигналы или сообщения иногда отправляются в виде матрицы. Данные могут быть расшифрованы только с помощью обратимой матрицы и определителя. В наших целях мы ориентируемся на определитель как на показатель обратимости матрицы.Вычисление определителя матрицы требует следования определенным шаблонам, описанным в этом разделе.

Общее примечание: Найдите определитель матрицы 2 × 2

Определитель матрицы [latex] 2 \ text {} \ times \ text {} 2 [/ latex], учитывая

[латекс] A = \ left [\ begin {array} {cc} a & b \\ c & d \ end {array} \ right] [/ latex]

определяется как

Рисунок 1

Обратите внимание на изменение обозначений. Есть несколько способов указать определитель, включая [latex] \ mathrm {det} \ left (A \ right) [/ latex] и замену скобок в матрице прямыми линиями, [latex] | A | [/ latex] .

Пример 1: Нахождение определителя матрицы 2 × 2

Найдите определитель заданной матрицы.

[латекс] A = \ left [\ begin {array} {cc} 5 & 2 \\ -6 & 3 \ end {array} \ right] [/ latex]

Решение

[латекс] \ begin {array} {l} \ mathrm {det} \ left (A \ right) = | \ begin {array} {cc} 5 & 2 \\ -6 & 3 \ end {array} | \ hfill \ \ = 5 \ left (3 \ right) – \ left (-6 \ right) \ left (2 \ right) \ hfill \\ = 27 \ hfill \ end {array} [/ latex]

Использование правила Крамера для решения системы двух уравнений с двумя переменными

Теперь мы представим последний метод решения систем уравнений, использующий определители.Этот метод, известный как правило Крамера , восходит к середине 18 века и назван в честь своего новатора, швейцарского математика Габриэля Крамера (1704–1752 гг.), Который представил его в 1750 году во Введении к анализу линий Курб. algébriques. Правило Крамера – это жизнеспособный и эффективный метод поиска решений систем с произвольным числом неизвестных при условии, что у нас есть такое же количество уравнений, что и неизвестных.

Правило Крамера даст нам единственное решение системы уравнений, если оно существует.Однако, если система не имеет решения или бесконечное количество решений, это будет обозначено нулевым определителем. Чтобы выяснить, является ли система непоследовательной или зависимой, необходимо использовать другой метод, например исключение.

Чтобы понять правило Крамера, давайте внимательно рассмотрим, как мы решаем системы линейных уравнений с использованием основных операций со строками. Рассмотрим систему двух уравнений с двумя переменными.

[латекс] \ begin {array} {c} {a} _ {1} x + {b} _ {1} y = {c} _ {1} \ left (1 \ right) \\ {a} _ { 2} x + {b} _ {2} y = {c} _ {2} \ left (2 \ right) \ end {array} [/ latex]

Мы исключаем одну переменную, используя операции со строками, и решаем для другой.Скажите, что мы хотим решить для [latex] x [/ latex]. Если уравнение (2) умножается на коэффициент, противоположный коэффициенту [латекс] y [/ латекс] в уравнении (1), уравнение (1) умножается на коэффициент [латекс] y [/ латекс] в уравнении (2) ), и мы добавляем два уравнения, переменная [latex] y [/ latex] будет удалена.

[латекс] \ begin {array} \ text {} b_ {2} a_ {1} x + b_ {2} b_ {1} y = b_ {2} c_ {1} \ hfill & \ text {Multiply} R_ { 1} \ text {by} b_ {2} \\ – b_ {1} a_ {2} x − b_ {1} b_ {2} y = −b_ {1} c_ {2} \ hfill & \ text {Умножить} R_ {2} \ text {by} −b_ {2} \\ \ text {______________________} \\ b_ {2} a_ {1} x − b_ {1} a_ {2} x = −b_ {2} c_ { 1} −b_ {1} c_ {2} \ end {array} [/ latex]

Теперь решите для [латекс] x [/ латекс].

[латекс] \ begin {array} {l} {b} _ {2} {a} _ {1} x- {b} _ {1} {a} _ {2} x = {b} _ {2 } {c} _ {1} – {b} _ {1} {c} _ {2} \ hfill \\ x \ left ({b} _ {2} {a} _ {1} – {b} _ {1} {a} _ {2} \ right) = {b} _ {2} {c} _ {1} – {b} _ {1} {c} _ {2} \ hfill \\ \ text { } x = \ frac {{b} _ {2} {c} _ {1} – {b} _ {1} {c} _ {2}} {{b} _ {2} {a} _ {1 } – {b} _ {1} {a} _ {2}} = \ frac {\ left [\ begin {array} {cc} {c} _ {1} & {b} _ {1} \\ { c} _ {2} & {b} _ {2} \ end {array} \ right]} {\ left [\ begin {array} {cc} {a} _ {1} & {b} _ {1} \\ {a} _ {2} & {b} _ {2} \ end {array} \ right]} \ hfill \ end {array} [/ latex]

Аналогичным образом, чтобы найти [latex] y [/ latex], мы исключим [latex] x [/ latex].

[латекс] \ begin {array} \ text {} a_ {2} a_ {1} x + a_ {2} b_ {1} y = a_ {2} c_ {1} \ hfill & \ text {Multiply} R_ { 1} \ text {by} a_ {2} \\ – a_ {1} a_ {2} x − a_ {1} b_ {2} y = −a_ {1} c_ {2} \ hfill & \ text {Умножить} R_ {2} \ text {by} −a_ {1} \\ \ text {______________________} \\ a_ {2} b_ {1} y − a_ {1} b_ {2} y = a_ {2} c_ {1 } −a_ {1} c_ {2} \ end {array} [/ latex]

Решение для [latex] y [/ latex] дает

[латекс] \ begin {array} {l} {a} _ {2} {b} _ {1} y- {a} _ {1} {b} _ {2} y = {a} _ {2 } {c} _ {1} – {a} _ {1} {c} _ {2} \ hfill \\ y \ left ({a} _ {2} {b} _ {1} – {a} _ {1} {b} _ {2} \ right) = {a} _ {2} {c} _ {1} – {a} _ {1} {c} _ {2} \ hfill \\ \ text { } y = \ frac {{a} _ {2} {c} _ {1} – {a} _ {1} {c} _ {2}} {{a} _ {2} {b} _ {1 } – {a} _ {1} {b} _ {2}} = \ frac {{a} _ {1} {c} _ {2} – {a} _ {2} {c} _ {1} } {{a} _ {1} {b} _ {2} – {a} _ {2} {b} _ {1}} = \ frac {| \ begin {array} {cc} {a} _ { 1} & {c} _ {1} \\ {a} _ {2} & {c} _ {2} \ end {array} |} {| \ begin {array} {cc} {a} _ {1 } & {b} _ {1} \\ {a} _ {2} & {b} _ {2} \ end {array} |} \ hfill \ end {array} [/ latex]

Обратите внимание, что знаменатель для [latex] x [/ latex] и [latex] y [/ latex] является определителем матрицы коэффициентов.

Мы можем использовать эти формулы для решения для [latex] x [/ latex] и [latex] y [/ latex], но правило Крамера также вводит новые обозначения:

Ключом к правилу Крамера является замена интересующего столбца переменных столбцом констант и вычисление детерминантов. Затем мы можем выразить [латекс] x [/ latex] и [latex] y [/ latex] как частное двух определителей.

Общее примечание: правило Крамера для систем 2 × 2

Правило Крамера – это метод, который использует детерминанты для решения систем уравнений, которые имеют то же количество уравнений, что и переменные.

Рассмотрим систему двух линейных уравнений с двумя переменными.

[латекс] \ begin {array} {c} {a} _ {1} x + {b} _ {1} y = {c} _ {1} \\ {a} _ {2} x + {b} _ {2} y = {c} _ {2} \ end {array} [/ latex]

Решение, использующее правило Крамера, дается как

[латекс] x = \ frac {{D} _ {x}} {D} = \ frac {| \ begin {array} {cc} {c} _ {1} & {b} _ {1} \\ {c} _ {2} & {b} _ {2} \ end {array} |} {| \ begin {array} {cc} {a} _ {1} & {b} _ {1} \\ { a} _ {2} & {b} _ {2} \ end {array} |}, D \ ne 0; \ text {} \ text {} y = \ frac {{D} _ {y}} {D } = \ frac {| \ begin {array} {cc} {a} _ {1} & {c} _ {1} \\ {a} _ {2} & {c} _ {2} \ end {array } |} {| \ begin {array} {cc} {a} _ {1} & {b} _ {1} \\ {a} _ {2} & {b} _ {2} \ end {array} |}, D \ ne 0 [/ латекс].

Если мы решаем для [latex] x [/ latex], столбец [latex] x [/ latex] заменяется столбцом констант. Если мы решаем для [latex] y [/ latex], столбец [latex] y [/ latex] заменяется постоянным столбцом.

Пример 2: Использование правила Крамера для решения системы 2 × 2

Решите следующую систему [latex] 2 \ text {} \ times \ text {} 2 [/ latex], используя правило Крамера.

[латекс] \ begin {array} {c} 12x + 3y = 15 \\ \ text {} 2x – 3y = 13 \ end {array} [/ latex]

Решение

Решите для [латекс] x [/ латекс].

[латекс] x = \ frac {{D} _ {x}} {D} = \ frac {| \ begin {array} {rr} \ hfill 15 & \ hfill 3 \\ \ hfill 13 & \ hfill -3 \ end {array} |} {| \ begin {array} {rr} \ hfill 12 & \ hfill 3 \\ \ hfill 2 & \ hfill -3 \ end {array} |} = \ frac {-45 – 39} {- 36 – 6} = \ frac {-84} {- 42} = 2 [/ latex]

Найдите [латекс] и [/ латекс].

[латекс] y = \ frac {{D} _ {y}} {D} = \ frac {| \ begin {array} {rr} \ hfill 12 & \ hfill 15 \\ \ hfill 2 & \ hfill 13 \ end { array} |} {| \ begin {array} {rr} \ hfill 12 & \ hfill 3 \\ \ hfill 2 & \ hfill -3 \ end {array} |} = \ frac {156 – 30} {- 36 – 6} = – \ frac {126} {42} = – 3 [/ латекс]

Решение [латекс] \ left (2, -3 \ right) [/ latex].

Попробуй 1

Используйте правило Крамера для решения системы уравнений 2 × 2.

[латекс] \ begin {массив} {l} \ text {} x + 2y = -11 \ hfill \\ -2x + y = -13 \ hfill \ end {array} [/ latex]

Использование правила Крамера для решения системы трех уравнений с тремя переменными

Вычисление определителя матрицы 3 × 3

Найти определитель матрицы 2 × 2 несложно, но найти определитель матрицы 3 × 3 сложнее.Один из способов – увеличить матрицу 3 × 3 повторением первых двух столбцов, получив матрицу 3 × 5. Затем мы вычисляем сумму произведений записей на по каждой из трех диагоналей (от верхнего левого угла к нижнему правому) и вычитаем произведения записей на по каждой из трех диагоналей (от нижнего левого угла к верхнему правому). Это легче понять с помощью наглядного пособия и примера.

Найдите определитель матрицы 3 × 3.

[латекс] A = \ left [\ begin {array} {ccc} {a} _ {1} & {b} _ {1} & {c} _ {1} \\ {a} _ {2} & {b} _ {2} & {c} _ {2} \\ {a} _ {3} & {b} _ {3} & {c} _ {3} \ end {array} \ right] [/ латекс]

  1. Дополните [латекс] A [/ латекс] первыми двумя столбцами.

    [латекс] \ mathrm {det} \ left (A \ right) = | \ begin {array} {ccc} {a} _ {1} & {b} _ {1} & {c} _ {1} \ \ {a} _ {2} & {b} _ {2} & {c} _ {2} \\ {a} _ {3} & {b} _ {3} & {c} _ {3} \ end {array} | \ begin {array} {c} {a} _ {1} \\ {a} _ {2} \\ {a} _ {3} \ end {array} \ begin {array} {c } {b} _ {1} \\ {b} _ {2} \\ {b} _ {3} \ end {array} | [/ latex]

  2. С верхнего левого угла в нижний правый: умножение значений по первой диагонали. Добавьте результат к произведению записей по второй диагонали. Добавьте этот результат к произведению записей по третьей диагонали.
  3. От левого нижнего угла до правого верхнего: вычтите произведение входов вверх по первой диагонали. Из этого результата вычтите произведение входов вверх по второй диагонали. Из этого результата вычтите произведение входов до третьей диагонали.

Рисунок 2

Алгебра выглядит следующим образом:

[латекс] | A | = {a} _ {1} {b} _ {2} {c} _ {3} + {b} _ {1} {c} _ {2} {a} _ {3 } + {c} _ {1} {a} _ {2} {b} _ {3} – {a} _ {3} {b} _ {2} {c} _ {1} – {b} _ {3} {c} _ {2} {a} _ {1} – {c} _ {3} {a} _ {2} {b} _ {1} [/ latex]

Пример 3: Нахождение определителя матрицы 3 × 3

Найдите определитель матрицы 3 × 3 для данного

[латекс] A = \ left [\ begin {array} {ccc} 0 & 2 & 1 \\ 3 & -1 & 1 \\ 4 & 0 & 1 \ end {array} \ right] [/ latex]

Решение

Дополните матрицу первыми двумя столбцами, а затем следуйте формуле.Таким образом,

[латекс] \ begin {array} {l} | A | = | \ begin {array} {ccc} 0 & 2 & 1 \\ 3 & -1 & 1 \\ 4 & 0 & 1 \ end {array} | \ begin {array} {c} 0 \\ 3 \\ 4 \ end {array} \ begin {array} {c} 2 \\ -1 \\ 0 \ end {array} | \ hfill \\ = 0 \ left (-1 \ right ) \ влево (1 \ вправо) +2 \ влево (1 \ вправо) \ влево (4 \ вправо) +1 \ влево (3 \ вправо) \ влево (0 \ вправо) -4 \ влево (-1 \ вправо) \ left (1 \ right) -0 \ left (1 \ right) \ left (0 \ right) -1 \ left (3 \ right) \ left (2 \ right) \ hfill \\ = 0 + 8 + 0 + 4 – 0-6 \ hfill \\ = 6 \ hfill \ end {array} [/ latex]

Попробуй 2

Найдите определитель матрицы 3 × 3.

[латекс] \ mathrm {det} \ left (A \ right) = | \ begin {array} {ccc} 1 & -3 & 7 \\ 1 & 1 & 1 \\ 1 & -2 & 3 \ end {array} | [/ latex ]

Вопросы и ответы

Можем ли мы использовать тот же метод, чтобы найти определитель большей матрицы?

Нет, этот метод работает только для [latex] 2 \ text {} \ times \ text {} 2 [/ latex] и [latex] \ text {3} \ text {} \ times \ text {} 3 [/ латексные] матрицы. Для больших матриц лучше всего использовать графическую утилиту или компьютерное программное обеспечение.

Использование правила Крамера для решения системы трех уравнений с тремя переменными

Теперь, когда мы можем найти определитель матрицы 3 × 3, мы можем применить правило Крамера для решения системы трех уравнений с тремя переменными .Правило Крамера простое и следует шаблону, соответствующему правилу Крамера для матриц 2 × 2. Однако по мере увеличения порядка матрицы до 3 × 3 требуется гораздо больше вычислений.

Когда мы вычисляем, что определитель равен нулю, правило Крамера не дает никаких указаний на то, что у системы нет решения или есть бесконечное количество решений. Чтобы выяснить это, мы должны выполнить устранение в системе.

Рассмотрим систему уравнений 3 × 3.

Рисунок 3

[латекс] x = \ frac {{D} _ {x}} {D}, y = \ frac {{D} _ {y}} {D}, z = \ frac {{D} _ {z} } {D}, D \ ne 0 [/ латекс]

где

Рисунок 4

Если мы записываем определитель [latex] {D} _ {x} [/ latex], мы заменяем столбец [latex] x [/ latex] на столбец констант.Если мы пишем определитель [latex] {D} _ {y} [/ latex], мы заменяем столбец [latex] y [/ latex] постоянным столбцом. Если мы пишем определитель [latex] {D} _ {z} [/ latex], мы заменяем столбец [latex] z [/ latex] постоянным столбцом. Всегда проверяйте ответ.

Пример 4: Решение системы 3 × 3 с использованием правила Крамера

Найдите решение данной системы 3 × 3, используя правило Крамера.

[латекс] \ begin {array} {c} x + y-z = 6 \\ 3x – 2y + z = -5 \\ x + 3y – 2z = 14 \ end {array} [/ latex]

Решение

Используйте правило Крамера.

[латекс] D = | \ begin {array} {ccc} 1 & 1 & -1 \\ 3 & -2 & 1 \\ 1 & 3 & -2 \ end {array} |, {D} _ {x} = | \ begin { array} {ccc} 6 & 1 & -1 \\ -5 & -2 & 1 \\ 14 & 3 & -2 \ end {array} |, {D} _ {y} = | \ begin {array} {ccc} 1 & 6 & -1 \\ 3 & -5 & 1 \\ 1 & 14 & -2 \ end {array} |, {D} _ {z} = | \ begin {array} {ccc} 1 & 1 & 6 \\ 3 & -2 & -5 \\ 1 & 3 & 14 \ end {array} | [/ latex]

Затем,

[латекс] \ begin {array} {l} x = \ frac {{D} _ {x}} {D} = \ frac {-3} {- 3} = 1 \ hfill \\ y = \ frac { {D} _ {y}} {D} = \ frac {-9} {- 3} = 3 \ hfill \\ z = \ frac {{D} _ {z}} {D} = \ frac {6} {-3} = – 2 \ hfill \ end {array} [/ latex]

Решение [латекс] \ left (1,3, -2 \ right) [/ latex].

Попробовать 3

Используйте правило Крамера, чтобы решить матрицу 3 × 3.

[латекс] \ begin {array} {r} \ hfill x – 3y + 7z = 13 \\ \ hfill x + y + z = 1 \\ \ hfill x – 2y + 3z = 4 \ end {array} [/ латекс]

Пример 5: Использование правила Крамера для решения несовместимой системы

Решите систему уравнений, используя правило Крамера.

[латекс] \ begin {array} {l} 3x – 2y = 4 \ text {} \ left (1 \ right) \\ 6x – 4y = 0 \ text {} \ left (2 \ right) \ end {массив } [/ латекс]

Решение

Начнем с нахождения определителей [латекс] D, {D} _ {x}, \ text {и} {D} _ {y} [/ latex].

[латекс] D = | \ begin {array} {cc} 3 & -2 \\ 6 & -4 \ end {array} | = 3 \ left (-4 \ right) -6 \ left (-2 \ right) = 0 [/ латекс]

Мы знаем, что нулевой определитель означает, что либо система не имеет решения, либо имеет бесконечное количество решений. Чтобы узнать, какой из них, мы используем процесс исключения. Наша цель – исключить одну из переменных.

  1. Умножьте уравнение (1) на [латекс] -2 [/ латекс].
  2. Добавьте результат в уравнение [латекс] \ left (2 \ right) [/ latex].

[латекс] \ begin {array} \ text {} \ hfill − 6x + 4y = −8 \\ \ hfill6x − 4y = 0 \\ \ hfill \ text {_____________} \\ \ hfill 0 = 8 \ end { array} [/ latex]

Получаем уравнение [латекс] 0 = -8 [/ латекс], которое неверно.Следовательно, у системы нет решения. График системы показывает две параллельные линии.

Рисунок 5

Пример 6. Использование правила Крамера для решения зависимой системы

Решите систему с бесконечным количеством решений.

[латекс] \ begin {array} {rr} \ hfill x – 2y + 3z = 0 & \ hfill \ left (1 \ right) \\ \ hfill 3x + y – 2z = 0 & \ hfill \ left (2 \ right) \\ \ hfill 2x – 4y + 6z = 0 & \ hfill \ left (3 \ right) \ end {array} [/ latex]

Решение

Давайте сначала найдем определитель.Создайте матрицу, дополненную первыми двумя столбцами.

[латекс] | \ begin {array} {rrr} \ hfill 1 & \ hfill -2 & \ hfill 3 \\ \ hfill 3 & \ hfill 1 & \ hfill -2 \\ \ hfill 2 & \ hfill -4 & \ hfill 6 \ end { array} \ text {} | \ text {} \ begin {array} {rr} \ hfill 1 & \ hfill -2 \\ \ hfill 3 & \ hfill 1 \\ \ hfill 2 & \ hfill -4 \ end {array} | [ / латекс]

Затем,

[латекс] 1 \ влево (1 \ вправо) \ влево (6 \ вправо) + \ влево (-2 \ вправо) \ влево (-2 \ вправо) \ влево (2 \ вправо) +3 \ влево (3 \ вправо) \ влево (-4 \ вправо) -2 \ влево (1 \ вправо) \ влево (3 \ вправо) – \ влево (-4 \ вправо) \ влево (-2 \ вправо) \ влево (1 \ вправо) -6 \ left (3 \ right) \ left (-2 \ right) = 0 [/ латекс]

Поскольку определитель равен нулю, решения либо нет, либо существует бесконечное количество решений.Чтобы выяснить это, нам нужно провести отбор.

  1. Умножьте уравнение (1) на [латекс] -2 [/ латекс] и добавьте результат к уравнению (3):

    [латекс] \ frac {\ begin {array} {r} \ hfill -2x + 4y – 6x = 0 \\ \ hfill 2x – 4y + 6z = 0 \ end {array}} {0 = 0} [/ latex ]

  2. Получение ответа [latex] 0 = 0 [/ latex], утверждение, которое всегда верно, означает, что система имеет бесконечное количество решений. Изобразив систему, мы можем увидеть, что две плоскости одинаковы, и обе они пересекают третью плоскость по прямой.

Рисунок 6

Детерминанты и правило Крамера

Линейные системы двух переменных и правило Крамера

Напомним, что матрица – это прямоугольный массив чисел, состоящий из строк и столбцов. Классифицируем матрицы по количеству строк n и количеству столбцов m . Например, матрица 3 × 4, читаемая как «матрица 3 на 4», состоит из 3 строк и 4 столбцов. Квадратная матрица Матрица с одинаковым количеством строк и столбцов.- матрица, в которой количество строк совпадает с количеством столбцов. В этом разделе мы обрисовываем еще один метод решения линейных систем с использованием специальных свойств квадратных матриц. Начнем с рассмотрения следующей матрицы коэффициентов 2 × 2 A ,

A = [a1b1a2b2]

Определитель Действительное число, связанное с квадратной матрицей. матрицы 2 × 2, обозначенной вертикальными линиями | A |, или более компактно как det ( A ), определяется следующим образом:

Определитель – это действительное число, которое получается вычитанием произведений значений на диагонали.

Пример 1

Вычислить: | 3−52−2 |.

Решение:

Вертикальные линии по обе стороны от матрицы показывают, что нам нужно вычислить определитель.

| 3−52−2 | = 3 (−2) −2 (−5) = – 6 + 10 = 4

Ответ: 4

Пример 2

Вычислить: | −6403 |.

Решение:

Обратите внимание, что матрица представлена ​​в виде верхнего треугольника.

| −6403 | = −6 (3) −4 (0) = – 18−0 = −18

Ответ: −18

Мы можем решать линейные системы с двумя переменными, используя определители. Начнем с общей линейной системы 2 × 2 и решим относительно y . Чтобы исключить переменную x , умножьте первое уравнение на −a2, а второе уравнение на a1.

{a1x + b1y = c1 a2x + b2y = c2 ⇒ × (−a2) ⇒ × a1 {−a1a2x − a2b1y = −a2c1a1a2x + a1b2y = a1c2

Это приводит к эквивалентной линейной системе, в которой переменная x выровнена для исключения.Теперь складывая уравнения, мы получаем

И числитель, и знаменатель очень похожи на определитель матрицы 2 × 2. На самом деле это так. Знаменатель – это определитель матрицы коэффициентов. А числитель – это определитель матрицы, образованной заменой столбца, представляющего коэффициенты y , на соответствующий столбец констант. Эта специальная матрица обозначается Dy.

y = DyD = | a1c1a2c2 | | a1b1a2b2 | = a1c2 − a2c1a1b2 − a2b1

Аналогичным образом может быть получено значение x .

x = DxD = | c1b1c2b2 | | a1b1a2b2 | = c1b2 − c2b1a1b2 − a2b1

В целом, мы можем сформировать расширенную матрицу следующим образом:

{a1x + b1y = c1a2x + b2y = c2 ⇔ [a1b1 | c1a2b2 | c2]

, а затем определите D, Dx и Dy, вычислив следующие детерминанты.

D = | a1b1a2b2 | Dx = | c1b1c2b2 | Dy = | a1c1a2c2 |

Решение системы в терминах определителей, описанных выше, когда D ≠ 0, называется правилом Крамера: Решение независимой системы линейных уравнений, выраженное в терминах определителей..

Правило Крамера (x, y) = (DxD, DyD)

Эта теорема названа в честь Габриэля Крамера (1704 – 1752).

Рисунок 3.2

Габриэль Крамер

Шаги решения линейной системы с двумя переменными с использованием определителей (правило Крамера) описаны в следующем примере.

Пример 3

Решите, используя правило Крамера: {2x + y = 7 3x − 2y = −7.

Решение:

Перед началом этого процесса убедитесь, что линейная система имеет стандартную форму.

Шаг 1 : Постройте расширенную матрицу и сформируйте матрицы, используемые в правиле Крамера.

{2x + y = 7 3x − 2y = −7 ⇒ [21 | 73−2 | −7]

В квадратной матрице, используемой для определения Dx, замените первый столбец матрицы коэффициентов константами. В квадратной матрице, используемой для определения Dy, замените второй столбец константами.

D = | 213−2 | Dx = | 71−7−2 | Dy = | 273−7 |

Шаг 2 : Рассчитайте детерминанты.

Dx = | 71−7−2 | = 7 (−2) – (- 7) (1) = – 14 + 7 = −7Dy = | 273−7 | = 2 (−7) −3 (7) = −14−21 = −35D = | 213−2 | = 2 (−2) −3 (1) = – 4−3 = −7

Шаг 3 : Используйте правило Крамера для вычисления x и y .

x = DxD = -7-7 = 1 и y = DyD = -35-7 = 5

Следовательно, одновременное решение (x, y) = (1,5).

Шаг 4 : Проверка необязательна; однако мы делаем это здесь для полноты картины.

Чек: (1,5)

Уравнение 1

Уравнение 2

2x + y = 72 (1) + (5) = 72 + 5 = 77 = 7 ✓

3x − 2y = −73 (1) −2 (5) = – 73−10 = −7−7 = −7 ✓

Ответ: (1, 5)

Пример 4

Решите, используя правило Крамера: {3x − y = −26x + 4y = 2.

Решение:

Соответствующая расширенная матрица коэффициентов следует.

{3x − y = −26x + 4y = 2 ⇒ [3−1 | −264 | 2]

А у нас,

Dx = | −2−124 | = −8 – (- 2) = – 8 + 2 = −6Dy = | 3−262 | = 6 – (- 12) = 6 + 12 = 18D = | 3−164 | = 12 – (- 6) = 12 + 6 = 18

Используйте правило Крамера, чтобы найти решение.

x = DxD = -618 = -13 и y = DyD = 1818 = 1

Ответ: (−13,1)

Попробуй! Решите, используя правило Крамера: {5x − 3y = −7−7x + 6y = 11.

Ответ: (−1, 23)

Когда определитель матрицы коэффициентов D равен нулю, формулы правила Крамера не определены. В этом случае система либо зависима, либо несовместима в зависимости от значений Dx и Dy. Когда D = 0 и оба Dx = 0 и Dy = 0, система является зависимой. Когда D = 0 и либо Dx, либо Dy отличны от нуля, система несовместима.

Когда D = 0, Dx = 0 и Dy = 0 ⇒ Зависимая система Dx ≠ 0 или Dy ≠ 0 ⇒ Несогласованная система

Пример 5

Решите, используя правило Крамера: {x + 15y = 3 5x + y = 15.

Решение:

Соответствующая расширенная матрица следует.

{x + 15y = 3 5x + y = 15 ⇒ [115 | 351 | 15]

А имеем следующее.

Dx = | 315151 | = 3−3 = 0Dy = | 13515 | = 15−15 = 0D = | 11551 | = 1−1 = 0

Если мы попытаемся использовать правило Крамера, мы получим

x = DxD = 00 и y = DyD = 00

, оба из которых являются неопределенными количествами.Поскольку D = 0 и как Dx = 0, так и Dy = 0, мы знаем, что это зависимая система. Фактически, мы можем увидеть, что оба уравнения представляют одну и ту же линию, если мы решим относительно y .

{x + 15y = 3 5x + y = 15 ⇒ {y = −5x + 15 y = −5x + 15

Следовательно, мы можем представить все решения (x, −5x + 15), где x – действительное число.

Ответ: (x, −5x + 15)

Попробуй! Решите, используя правило Крамера: {3x − 2y = 10 6x − 4y = 12.

Ответ: Ø

Линейные системы трех переменных и правило Крамера

Рассмотрим следующую матрицу коэффициентов 3 × 3 A ,

A = [a1b1c1a2b2c2a3b3c3]

Определитель этой матрицы определяется следующим образом:

det (A) = | a1b1c1a2b2c2a3b3c3 | = a1 | b2c2b3c3 | −b1 | a2c2a3c3 | + c1 | a2b2a3b3 | = a1 (b2c3 − b3c2) −b1 (a2c3 − 9 −3 ab2) +

Здесь каждый определитель 2 × 2 называется второстепенным определителем матрицы, которая получается после удаления строки и столбца квадратной матрицы.предыдущего фактора. Обратите внимание, что множители – это элементы в первой строке матрицы, и что они меняют знак (+ – +).

Пример 6

Рассчитать: | 1322−1305−1 |.

Решение:

Чтобы легко определить второстепенное значение каждого фактора в первой строке, мы выстраиваем первую строку и соответствующий столбец. Определитель матрицы оставшихся элементов определяет соответствующий минор.

Позаботьтесь о том, чтобы поменять знак множителей в первой строке. Далее следует разложение несовершеннолетними о первом ряду:

| 1322−1305−1 | = 1 | −135−1 | −3 | 230−1 | +2 | 2−105 | = 1 (1−15) −3 (−2−0) +2 (10−0) = 1 (−14) −3 (−2) +2 (10) = – 14 + 6 + 20 = 12

Ответ: 12

Расширение по несовершеннолетним может производиться по любой строке или любому столбцу. Знак коэффициентов, определяемый выбранной строкой или столбцом, будет чередоваться в соответствии со следующим массивом знаков.

[+ – + – + – + – +]

Поэтому, чтобы расширить второй ряд, мы будем чередовать знаки, начиная с противоположного первого элемента. Мы можем расширить предыдущий пример о второй строке, чтобы показать, что получен такой же ответ для определителя.

А можно написать,

| 1322−1305−1 | = – (2) | 325−1 | + (- 1) | 120−1 | – (3) | 1305 | = −2 (−3−10) −1 (−1−0) −3 (5−0) = – 2 (−13) −1 (−1) −3 (5) = 26 + 1−15 = 12

Обратите внимание, что мы получаем тот же ответ 12.

Пример 7

Рассчитать: | 4306122410 |.

Решение:

Расчеты упрощаются, если мы расширим третий столбец, потому что он содержит два нуля.

Далее следует расширение несовершеннолетними по поводу третьего столбца:

| 4306122410 | = 0 | 61241 | −2 | 4341 | +0 | 43612 | = 0−2 (4−12) + 0 = −2 (−8) = 16

Ответ: 16

Следует отметить, что существуют и другие методы, используемые для запоминания того, как вычислить определитель матрицы 3 × 3.Кроме того, многие современные калькуляторы и системы компьютерной алгебры могут найти определитель матриц. Предлагаем вам изучить эту обширную тему.

Мы можем решить линейные системы с тремя переменными, используя определители. Для этого мы начнем с расширенной матрицы коэффициентов,

{a1x + b1y + c1z = d1a2x + b2y + c2z = d2a3x + b3y + c3z = d3 ⇔ [a1b1c1 | d1a2b2c2 | d2a3b3c3 | d3]

Пусть D представляет определитель матрицы коэффициентов,

D = | a1b1c1a2b2c2a3b3c3 |

Затем определите Dx, Dy и Dz, вычислив следующие определители.

Dx = | d1b1c1d2b2c2d3b3c3 | Dy = | a1d1c1a2d2c2a3d3c3 | Dz = | a1b1d1a2b2d2a3b3d3 |

Когда D ≠ 0, решение системы в терминах детерминантов, описанных выше, может быть вычислено с использованием правила Крамера:

Правило Крамера (x, y, z) = (DxD, DyD, DzD)

Используйте это для эффективного решения систем с тремя переменными.

Пример 8

Решите, используя правило Крамера: {3x + 7y − 4z = 02x + 5y − 3z = 1−5x + 2y + 4z = 8.

Решение:

Начните с определения соответствующей расширенной матрицы.

{3x + 7y − 4z = 02x + 5y − 3z = 1−5x + 2y + 4z = 8 ⇔ [37−4 | 025−3 | 1−524 | 8]

Затем вычислите определитель матрицы коэффициентов.

D = | 37−425−3−524 | = 3 | 5−324 | −7 | 2−3−54 | + (- 4) | 25−52 | = 3 (20 – (- 6)) – 7 (8−15) −4 (4 – (- 25)) = 3 (26) −7 (−7) −4 (29) = 78 + 49−116 = 11

Аналогичным образом мы можем вычислить Dx, Dy и Dz.Это оставлено как упражнение.

Dx = | 07−415−3824 | = −44Dy = | 30−421−3−584 | = 0Dz = | 370251−528 | = −33

Используя правило Крамера, мы имеем,

x = DxD = −4411 = −4 y = DyD = 011 = 0 z = DzD = −3311 = −3

Ответ: (−4,0, −3)

Если определитель матрицы коэффициентов D = 0, то система либо зависимая, либо противоречивая. Это будет зависеть от Dx, Dy и Dz. Если все они равны нулю, то система зависима.Если хотя бы один из них отличен от нуля, то он несовместим.

Когда D = 0, Dx = 0 и Dy = 0 и Dz = 0 ⇒ Зависимая система Dx ≠ 0 или Dy ≠ 0 или Dz ≠ 0 ⇒ Несогласованная система

Пример 9

Решите, используя правило Крамера: {4x − y + 3z = 521x − 4y + 18z = 7−9x + y − 9z = −8.

Решение:

Начните с определения соответствующей расширенной матрицы.

{4x − y + 3z = 521x − 4y + 18z = 7−9x + y − 9z = −8 ⇔ [4−13 | 521−418 | 7−91−9 | −8]

Затем определите определитель матрицы коэффициентов.

D = | 4−1321−418−91−9 | = 4 | −4181−9 | – (- 1) | 2118−9−9 | +3 | 21−4−91 | = 4 (36−18) +1 (−189 – (- 162)) + 3 (21−36) = 4 (18) +1 (−27) +3 (−15) = 72−27−45 = 0

Поскольку D = 0, система является либо зависимой, либо противоречивой.

Dx = | 5−137−418−81−9 | = 96

Однако, поскольку Dx отличен от нуля, мы заключаем, что система несовместима. Одновременного решения нет.

Ответ: Ø

Попробуй! Решите, используя правило Крамера: {2x + 6y + 7z = 4−3x − 4y + 5z = 125x + 10y − 3z = −13.

Ответ: (−3,12,1)

Основные выводы

  • Определитель матрицы – действительное число.
  • Определитель матрицы 2 × 2 получается вычитанием произведения значений на диагоналях.
  • Определитель матрицы 3 × 3 получается путем расширения матрицы с использованием миноров в любой строке или столбце. При этом позаботьтесь об использовании массива знаков для определения знака коэффициентов.
  • Используйте правило Крамера для эффективного определения решений линейных систем.
  • Когда определитель матрицы коэффициентов равен 0, правило Крамера не применяется; система будет либо зависимой, либо непоследовательной.

Тематические упражнения

    Часть A: Линейные системы с двумя переменными

      Вычислить определитель.

    1. | 1234 |

    2. | 5324 |

    3. | −13−3−2 |

    4. | 743−2 |

    5. | −41−30 |

    6. | 95−10 |

    7. | 1050 |

    8. | 0350 |

    9. | 04−13 |

    10. | 102102 |

    11. | a1b10b2 |

    12. | 0b1a2b2 |

      Решите, используя правило Крамера.

    1. {3x − 5y = 82x − 7y = 9

    2. {2x + 3y = −13x + 4y = −2

    3. {2x − y = −34x + 3y = 4

    4. {x + 3y = 15x − 6y = −9

    5. {х + у = 16х + 3у = 2

    6. {x − y = −15x + 10y = 4

    7. {5x − 7y = 144x − 3y = 6

    8. {9x + 5y = −97x + 2y = −7

    9. {6x − 9y = 3−2x + 3y = 1

    10. {3x − 9y = 32x − 6y = 2

    11. {4x − 5y = 203y = −9

    12. {x − y = 02x − 3y = 0

    13. {2x + y = ax + y = Ь

    14. {ax + y = 0by = 1

    Часть B: Линейные системы с тремя переменными

      Вычислить определитель.

    1. | 123213132 |

    2. | 251124323 |

    3. | −31−13−1−2−251 |

    4. | 1−15−45−1−12−3 |

    5. | 3−1223−1521 |

    6. | 40−33−100−52 |

    7. | 0−34−30602−3 |

    8. | 6−1−325284−1 |

    9. | 257035004 |

    10. | 210

    11. 004 |

    12. | a1b1c10b2c200c3 |

    13. | a100a2b20a3b3c3 |

      Решите, используя правило Крамера.

    1. {x − y + 2z = −33x + 2y − z = 13−4x − 3y + z = −18

    2. {3x + 4y − z = 104x + 6y + 7z = 92x + 3y + 5z = 3

    3. {5x + y − z = 02x − 2y + z = −9−6x − 5y + 3z = −13

    4. {−4x + 5y + 2z = 123x − y − z = −25x + 3y − 2z = 5

    5. {x − y + z = −1−2x + 4y − 3z = 43x − 3y − 2z = 2

    6. {2x + y − 4z = 72x − 3y + 2z = −44x − 5y + 2z = −5

    7. {4x + 3y − 2z = 22x + 5y + 8z = −1x − y − 5z = 3

    8. {x − y + z = 7x + 2y + z = 1x − 2y − 2z = 9

    9. {3x − 6y + 2z = 12−5x − 2y + 3z = 47x + 3y − 4z = −6

    10. {2x − y − 5z = 23x + 2y − 4z = −35x + y − 9z = 4

    11. {4x + 3y − 4z = −132x + 6y − 5z = −2−2x − 3y + 3z = 5

    12. {x − 2y + z = −14y − 3z = 03y − 2z = 1

    13. {2x + 3y − z = −5x + 2y = 03x + 10y = 4

    14. {2x − 3y − 2y = 9−3x + 4y + 4z = −13x − y − 2z = 4

    15. {2x + y − 2z = −1x − y + 3z = 23x + y − z = 1

    16. {3x − 8y + 9z = −2 − x + 5y − 10z = 3x − 3y + 4z = −1

    17. {5x − 6y + 3z = 23x − 4y + 2z = 02x − 2y + z = 0

    18. {5x + 10y − 4z = 122x + 5y + 4z = 0x + 5y − 8z = 6

    19. {5x + 6y + 7z = 22y + 3z = 34z = 4

    20. {x + 2z = −1−5y + 3z = 104x − 3y = 2

    21. {x + y + z = ax + 2y + 2z = a + bx + 2y + 3z = a + b + c

    22. {x + y + z = a + b + cx + 2y + 2z = a + 2b + 2cx + y + 2z = a + b + 2c

    Часть C: Обсуждение

    1. Изучите и обсудите историю детерминанта.Кто первым ввел обозначение определителя?

    2. Изучите другие способы вычисления определителя матрицы 3 × 3. Привести пример.

ответов

  1. (-12,2)

  2. (-13,43)

  3. (54, −3)

  1. (12,12, −1)

  2. (12z − 4,23z + 1, z)

  3. (-12,5,52)

Правило Крамера

Рассмотрим общую линейную систему 2 на 2

Умножение первого уравнения на a 22 , второго на – a 12 и сложение результатов исключает y и позволяет оценить x :

при условии, что a 11 a 22 a 12 a 21 ≠ 0.Точно так же умножение первого уравнения на – a 21 , второго на a 11 и сложение результатов исключает x и определяет y :

снова предполагая, что a 11 a 22 a 12 a 21 ≠ 0. Эти выражения для x и y могут быть записаны в терминах определителей как следует:

и

Если исходная система записана в матричной форме,

, то знаменатели в приведенных выше выражениях для неизвестных x и y оба равны определителю матрицы коэффициентов.Кроме того, числитель в выражении для первого неизвестного, x , равен определителю матрицы, которая получается, когда первый столбец матрицы коэффициентов заменяется столбцом констант, а числитель в выражении для второй неизвестный, y , равен определителю матрицы, которая получается, когда второй столбец матрицы коэффициентов заменяется столбцом констант. Это правило Крамера для линейной системы 2 на 2.

Расширение шаблона до линейной системы 3 на 3,

Правило Крамера гласит, что если определитель матрицы коэффициентов отличен от нуля, то выражения для неизвестных x, y и z принимают следующий вид:

Общая форма правила Крамера выглядит следующим образом: Система линейных уравнений n с неизвестными n , записанными в матричной форме A x = b как

будет иметь уникальное решение, если det A ≠ 0, и в этом случае значение неизвестного x j дается выражением

, где A j – это матрица, которая получается, когда столбец j матрицы коэффициентов A заменяется матрицей столбцов b .

Два важных теоретических результата о квадратных системах вытекают из правила Крамера:

Теорема F . Система квадратов A x = b будет иметь уникальное решение для каждой матрицы столбцов b тогда и только тогда, когда det A ≠ 0.

Теорема G . Однородная квадратная система A x = 0 будет иметь только тривиальное решение x = 0 тогда и только тогда, когда det A ≠ 0.

Хотя правило Крамера имеет теоретическое значение, поскольку дает формулу для неизвестных, обычно это неэффективный метод решения, особенно для больших систем. Метод исключения Гаусса по-прежнему остается предпочтительным. Однако правило Крамера может быть полезно, когда, например, необходимо значение только одного неизвестного.

Пример 1 : Используйте правило Крамера, чтобы найти значение y , учитывая, что

Поскольку эта линейная система эквивалентна матричному уравнению

Правило Крамера подразумевает, что второе неизвестное, y , дается выражением

при условии, что знаменатель – определитель матрицы коэффициентов – не равен нулю.Сокращение строк с последующим расширением Лапласа по первому столбцу оценивает эти детерминанты:

При этих расчетах (*) означает

Правило Крамера: метод определителей

Содержание | Дом

Урок 35

Одновременные уравнения: Раздел 2

Вернуться в раздел 1

Правило Крамера: метод определителей

Пример 4.Решите эту систему одновременных уравнений:

1) 3 x + 4 y = 19
2) 2 x y = 9

Решение .Если мы сложим уравнения как есть, ни одно из неизвестных не сократится. Теперь, если бы коэффициент y в уравнении 2) был -4, тогда y были бы сокращены. Поэтому мы расширим нашу стратегию следующим образом:

Сделайте одну пару коэффициентов отрицательными друг друга – на , умножив
обеих частей уравнения на одно и то же число. После добавления уравнений это неизвестное будет удалено.

Чтобы получить коэффициенты 4 и −4 при y , умножим обе части уравнения 2) на 4:

.
1) 3 x + 4 y = 19 3 x + 4 y = 19
2) 2 x y = 9 8 x 4 y = 36
11 x = 55
x = 55
11
x = 5

Число 4 над стрелкой в ​​уравнении 2) означает, что обе части этого уравнения были умножены на 4.Уравнение 1) не изменилось.

Чтобы найти y , подставьте x = 5 в любое из исходных уравнений. В уравнении 1):

3 · 5 + 4 y = 19
4 y = 19–15
4 y = 4
y = 1

Решение (5, 1).

Студент всегда должен проверять решение, заменяя x и y на (5, 1) в исходных уравнениях.

Пример 5. Решить одновременно:

1) 3 x + 2 y = -2
2) 2 x + 5 y = −5

Решение .Мы должны сделать одну пару коэффициентов отрицательной по отношению друг к другу. В этом примере мы должны решить, какие из неизвестных нужно исключить: x или y . В любом случае мы сделаем новые коэффициенты наименьшим общим кратным (НОК) исходных коэффициентов, но с противоположными знаками.

Таким образом, если мы исключим x , мы получим новые коэффициенты 6 и −6. (НОК 3 и 2 равно 6.) Хотя, если мы исключим и , мы сделаем их новые коэффициенты 10 и -10.(НОК 2 и 5 равно 10.)

Давайте выберем исключение x :

1) 3 x + 2 y = -2 6 x + 4 y = −4
2) 2 x + 5 y = −5 −6 х 15 y = 15
________________________________________________________________________
11 y = 11
y = -1.

Уравнение 1) было умножено на 2. Уравнение 2) было умножено на −3 – потому что мы хотим сделать эти коэффициенты 6 и −6, так что при сложении y отменяется.

Чтобы найти x , подставим y = −1 в исходное уравнение 1):

3 х + 2 (-1) = -2
3 x – 2 = -2
3 x = 0
x = 0

Решение (0, −1).

Проблема 3. Решайте одновременно.

1) 2 х + 3 y = 13
2) 5 х y = 7

Чтобы отменить y , умножьте уравнение 2) на 3:

.
1) 2 x + 3 y = 13 2 x + 3 y = 13
2) 5 x y = 7 15 x 3 y = 21
________________________________________________________________________
17 x = 34
x = 2

Чтобы найти для y :

Подставьте x = 2 в одно из исходных уравнений.
В уравнении 1:

2 · 2 + 3 y = 13
4 + 3 y = 13
3 y = 9
y = 3

Решение (2, 3).

Проблема 4. Решайте одновременно.

1) x + 2 y = -1
2) 2 х 3 y = 5

Чтобы уменьшить x , умножьте уравнение 1) на −2:

1) x + 2 y = -1 −2 x 4 y = 2
2) 2 x 3 y = 5 2 x 3 y = 5
________________________________________________________________________
7 y = 7
y = -1

Чтобы найти x :

Подставим y = −1 в одно из исходных уравнений.
В уравнении 1:

х + 2 (-1) = -1
x – 2 = -1
x = -1 + 2
x = 1

Решение (1, −1).

Мы могли бы исключить и , умножив уравнение 1) на 3 и уравнение 2) на 2.

Задача 5. Решить одновременно:

1) 3 х 4 y = 1
2) 2 х + 3 y = 12

Чтобы отменить y :

Умножьте уравнение 1) на 3 и уравнение 2) на 4:

.
1) 3 x 4 y = 1 9 x 12 y = 3
2) 2 x + 3 y = 12 8 x + 12 y = 48
________________________________________________________________________
17 x = 51
x = 51
17
x = 3

Чтобы найти для y :

Подставьте x = 3 в одно из исходных уравнений.
В уравнении 2 (поскольку знак y уже положительный):

2 · 3 + 3 y = 12
6 + 3 y = 12
3 y = 6
y = 2

Решение (3, 2).

Задача 6. Решить одновременно:

1) 3 х + 2 y = −4
2) 2 х + 5 y = 1

Чтобы отменить x :

Умножьте уравнение 1) на 2 и уравнение 2) на −3:

.
1) 3 x + 2 y = −4 6 x + 4 y = −8
2) 2 x + 5 y = 1 −6 x 15 y = −3
________________________________________________________________________
11 y = −11
y = 1

Чтобы найти x :

Подставьте y = 1 в одно из исходных уравнений.
В уравнении 1:

3 x + 2 · 1 = −4
3 x + 2 = −4
3 x = −4-2
3 x = −6
x = -2

Решение: (−2, 1).

Мы могли бы исключить y , умножив уравнение 1) на 5 и уравнение 2) на −2.

Задача 7. Решить одновременно:

1) 5 х + 3 y = −11
2) 2 х + 4 y = −10

Чтобы отменить x :

Умножьте уравнение 1) на 2 и уравнение 2) на −5:

.
1) 5 x + 3 y = −11 10 x + 6 y = −22
2) 2 x + 4 y = −10 −10 x 20 y = 50
________________________________________________________________________
14 y = 28
y = -2

Чтобы найти x :

Подставим y = −2 в одно из исходных уравнений.
В уравнении 1:

5 х + 3 (-2) = −11
5 x – 6 = −11
5 x = −11 + 6
5 x = −5
x = -1

Решение (−1, −2).

Мы могли бы исключить y , умножив уравнение 1) на 4 и уравнение 2) на −3.

Правило Крамера: метод определителей

Система двух уравнений с двумя неизвестными имеет следующий вид:

a – это коэффициенты x . b являются коэффициентами y . Ниже приводится матрица этих коэффициентов.

Число a 1 b 2 b 1 a 2 называется определителем этой матрицы.

дет = a 1 b 2 b 1 a 2

Обозначим этот определитель через D.

Теперь рассмотрим эту матрицу, в которой c заменяют коэффициенты x :

Тогда определитель этой матрицы, которую мы назовем D x , равен

.

c 1 b 2 b 1 c 2

И рассмотрим эту матрицу, в которой c заменяют коэффициенты y :

Определитель этой матрицы – D y – равен

.

a 1 c 2 c 1 a 2

Правило Крамера гласит следующее:

В каждой системе двух уравнений с двумя неизвестными
, в которой определитель D не равен 0,

x = D x
D
y = D y
D

Пример.Используйте правило Крамера для решения этой системы уравнений (Задача 7):

5 x + 3 y = −11
2 x + 4 y = −10

Решение .

D = дет = 5 · 4 – 3 · 2
= 20–6
= 14.
D x = дет = −11 · 4 – 3 · −10
= −44 + 30
= −14.
D y = дет = 5 · −10 – (−11) · 2
= −50 + 22
= −28.

Следовательно,

x = D x
D
= −14
14
= -1.
y = D y
D
= −28
14
= -2.

Проблема. Используйте правило Крамера для решения этих одновременных уравнений.

3 x 5 y = −31
2 x + y = 1
D = дет = 3 · 1 – (−5) · 2
= 3 + 10
= 13.
D x = дет = −31 · 1 – (−5) · 1
= −31 + 5
= −26.
D y = дет = 3 · 1 – (−31) · 2
= 3 + 62
= 65.

Следовательно,

x = D x
D
= −26
13
= -2.
y = D y
D
= 65
13
= 5.

Когда определитель D не равен 0, мы говорим, что уравнения линейно независимы . В любой системе линейно независимых уравнений есть одно-единственное решение.

Когда определитель D равен 0, то либо 1) нет единственного решения, можно назвать много; или 2) решения нет вообще. В случае 1) уравнения линейно зависимы . Один из них просто кратен другому.Например,

x + y = 3

2 x + 2 y = 6.

В случае 2) уравнения несовместимы .

x + y = 3

x + y = 4.

Раздел 3: Три уравнения с тремя неизвестными

Вернуться в раздел 1

Следующий урок: задачи со словами, которые приводят к одновременным уравнениям

Содержание | Дом


Сделайте пожертвование, чтобы TheMathPage оставалась в сети.
Даже 1 доллар поможет.


Авторские права © 2021 Лоуренс Спектор

Вопросы или комментарии?

Эл. Почта: [email protected]


Инверсия матрицы

и правило Крамера

1.3 Решение систем линейных уравнений: обращение матриц и правило Крамера

Два уравнения с двумя переменными

Рассмотрим систему двух уравнений с двумя переменными x 1 и x 2 :

a x 1 + b x 2 = u
c x 1 + d x 2 = v ,
где a , b , c , d , u и v – числа с a , b , c и d ненулевые.

Один из простых способов найти x 1 и x 2 – выделить одну из переменных в одном из уравнений и подставить результат в другое уравнение. Например, из второго уравнения имеем

x 2 = ( v c x 1 ) / d .

Подставляя это выражение для x 2 в первое уравнение, получаем

a x 1 + b ( v c x 1 ) / d = u ,

который мы можем записать как

( a b c / d ) x 1 + b v / d = u .

Сначала предположим, что a b c / d . потом или же Чтобы найти x 2 , мы используем тот факт, что x 2 = ( v c x 1 ) / d , чтобы получить

Теперь предположим, что a = b c / d . Тогда уравнения имеют вид

a x 1 + b x 2 = u
( a d / b ) x 1 + d x 2 = v
или, умножив первое уравнение на d , а второе на b ,
a d x 1 + b d x 2 = u d
a d x 1 + b d x 2 = v b .
Таким образом, в этом случае, если u d v b , то уравнения не имеют решения, и если u d = v b (в этом случае также v a = u c ), то набор решений – это набор пар ( x 1 , x 2 ), удовлетворяющих a x 1 + b x 2 = u (то есть набор пар ( x 1 , ( u a x 1 ) / b ) для любого числа x 1 ).

Таким образом, решения системы уравнений имеют три возможных вида.

  • Если a d b c , то уравнения имеют единственное решение,
    ( x 1 , x 2 ) =,.
  • Если a d = b c и u d = v b , то набором решений уравнений является набор пар
    x 1 ,
    для любого числа x 1 .
  • Если a d = b c и u d v b , то уравнения не имеют решения.

Этот метод выделения переменной в одном уравнении и подстановки ее в другое уравнение является громоздким, когда система состоит из более чем двух уравнений и двух переменных. Теперь я опишу более элегантный метод.

n уравнений в n переменных

Мы можем записать систему двух уравнений с двумя переменными из предыдущего раздела в матричной форме, как

Общая система уравнений n в переменных n может быть записана как

A x = b ,

где A – это матрица n × n , а x и b – векторы-столбцы n × 1.

Если A неособое число, то умножение каждой стороны на обратное A −1 A дает

x = A -1 b ,

единственное решение в этом случае. Таким образом, решение системы уравнений в данном случае сводится к нахождению обратной матрицы A .

Если определитель A равен нулю, то будет ли система уравнений иметь много решений или ни одного, зависит от рангов A и расширенной матрицы, определенной следующим образом.

Определение
Пусть A – это матрица n × n , а b – вектор-столбец n × 1. Расширенная матрица ( A , b ) – это матрица с n строк и n + 1 столбцами, в которой A составляет первые n строк и столбцов, а b – последний столбец. .
Следующий результат дает условия, при которых система уравнений A x = b имеет одно, много или ни одного решения.
Источник предложения
Пусть A будет матрицей n × n и пусть b будет вектором-столбцом n × 1. Если A невырожден, то система уравнений

A x = b

имеет уникальное решение: x = A −1 b . Если определитель A равен нулю, то система имеет бесконечно много решений, если ранг расширенной матрицы ( A , b ) равен рангу A , и в противном случае не имеет решения.
Источник скрыть
Для доказательства см. Hadley (1961), pp. 168–169.

Если n = 2 или n = 3, обратное к A относительно легко вычислить, так что, если система уравнений имеет решение, это решение может быть легко найдено, как показывают следующие примеры.

Пример
Рассмотрим систему уравнений
a x 1 + b x 2 = u
c x 1 + d x 2 = против .
Запишите это в матричной форме. Если a d b c , тогда существует обратная матрица слева и
д б
в a
так что у нас есть Таким образом а также как мы обнаружили раньше.
Пример
Рассмотрим систему уравнений
2 x 1 + x 2 + 2 x 3 = 1
x 1 x 2 + x 3 = 0
2 x 2 x 3 = 3.
При записи в матричной форме A x = b матрица A будет
2 1 2
1 -1 1
0 2 -1
.
Определитель этой матрицы равен 3 ≠ 0, поэтому матрица невырожденная. Используя общую формулу для обратной матрицы, обратная матрица равна
-1/3 5/3 1
1/3 -2/3 0
2/3 −4/3 -1
.
Умножение этой матрицы на транспонирование (1, 0, 3) дает ( x 1 , x 2 , x 3 ) = (8/3, 1/3, −7 / 3), что, таким образом, является единственным решением системы уравнений. (Подставляя этот список значений в исходные уравнения, вы можете убедиться, что это действительно решение.)
Пример
Рассмотрим систему уравнений
4 x 1 + 2 x 2 + x 3 = 1
2 x 1 2 x 2 + 2 x 3 = 0
4 x 2 2 x 3 = 3.
При записи в матричной форме матрица слева выглядит так:
4 2 1
2 -2 2
0 4 -2
.
Определитель этой матрицы равен 0; его ранг 2.Расширенная матрица
4 2 1 1
2 -2 2 0
0 4 -2 3
.
Эта матрица имеет ранг 3 (определитель матрицы 3 × 3, состоящей из последних трех строк и столбцов, равен 14 ≠ 0), поэтому согласно предыдущему результату система уравнений не имеет решения.(Другой способ убедиться, что система не имеет решения, – это умножить второе уравнение на 2, а третье уравнение на 1,5 и прибавить их вместе. Полученное уравнение несовместимо с первым уравнением.)
Пример
Рассмотрим систему уравнений
3 x 1 + 4 x 2 = 7
6 x 1 + 8 x 2 = 14.
При записи в матричной форме матрица слева выглядит так:
3 4
6 8
.
Определитель этой матрицы равен 0; ее ранг равен 1. Расширенная матрица
3 4 7
6 8 14
.
Эта матрица имеет ранг 1 (определитель каждой подматрицы 2 × 2 равен 0), поэтому согласно предыдущему результату система уравнений имеет бесконечно много решений.

Второе уравнение вдвое больше первого, поэтому любое ( x 1 , x 2 ), удовлетворяющее первому уравнению, удовлетворяет второму. Следовательно ( x 1 , x 2 ) является решением системы уравнений тогда и только тогда, когда 3 x 1 + 4 x 2 = 7, или, если вы например, x 2 = (7 – 3 x 1 ) / 4.

Правило Крамера

Полезное следствие того факта, что решение системы A x = b дается как x = A −1 b , если A неособое, следующий результат ( из-за Габриэля Крамера, 1704-1752), который дает явное выражение для значения каждой переменной в отдельности.
Предложение (правило Крамера)
Пусть A будет матрицей n × n , пусть b будет вектор-столбцом n × 1, и рассмотрим систему уравнений

A x = b

где x – вектор-столбец n × 1.Если A неособое, тогда (уникальное) значение x , которое удовлетворяет системе, определяется как

x i = | A * ( b , i ) | / | A | для i = 1, …, n ,

где A * ( b , i ) – это матрица, полученная из A заменой i -го столбца на b .
Проба
Имеем x = A −1 b , так что

x i = ∑ n
j = 1 v i j b j ,

где v i j – ( i , j ) -й компонент A −1 .

Теперь, согласно предыдущему результату, ( i , j ) -й компонент A −1 равен (−1) i + j | A j i | / | A |. Таким образом

x i = ∑ n
j = 1 (−1) i + j | A j i | b j / | A |.

Наконец, вычислите определитель A * ( b , i ) в выражении результата, развернув его по его i -му столбцу. Это столбец b , поэтому согласно второй части предыдущего результата мы имеем

| A * ( b , i ) | = ∑ n
j = 1 (−1) i + j b j | A j i |,

установление результата.(Обратите внимание, что, поскольку i является индексом столбца , по которому мы расширяемся, роли i и j здесь поменялись местами относительно их ролей в формулировке результата, который мы используем.)
Пример
Применение правила Крамера к системе
a x 1 + b x 2 = u
c x 1 + d x 2 = v ,
если a d b c получаем
x 1 =
u б
v д
a d b c
а также
x 2 =
a u
c в
a d b c
.
Вычисляя определители в числителях, имеем а также
Правило Крамера особенно полезно, если вы хотите вычислить значение только некоторых переменных в решении системы уравнений, как показано в следующем примере.
Пример
Значение x 2 в решении системы уравнений
2 x 1 + x 2 + 2 x 3 = 1
x 1 x 2 + x 3 = 0
2 x 2 x 3 = 3
является, по правилу Крамера, определителем
2 1 2
1 0 1
0 3 -1
делится на определитель
2 1 2
1 -1 1
0 2 -1
.

Оставить комментарий